Programs & Examples On #Checkmark

make UITableViewCell selectable only while editing

Have you tried setting the selection properties of your tableView like this:

tableView.allowsMultipleSelection = NO; tableView.allowsMultipleSelectionDuringEditing = YES; tableView.allowsSelection = NO; tableView.allowsSelectionDuringEditing YES; 

If you want more fine-grain control over when selection is allowed you can override - (NSIndexPath *)tableView:(UITableView *)tableView willSelectRowAtIndexPath:(NSIndexPath *)indexPath in your UITableView delegate. The documentation states:

Return Value An index-path object that confirms or alters the selected row. Return an NSIndexPath object other than indexPath if you want another cell to be selected. Return nil if you don't want the row selected. 

You can have this method return nil in cases where you don't want the selection to happen.

How to draw checkbox or tick mark in GitHub Markdown table?

|checked|unchecked|crossed|
|---|---|---|
|✓|_|✗|

Where

? via HTML Entity Code
? via HTML Entity Code
_ via underscore character
and table via markdown table syntax.

No provider for TemplateRef! (NgIf ->TemplateRef)

You missed the * in front of NgIf (like we all have, dozens of times):

<div *ngIf="answer.accepted">&#10004;</div>

Without the *, Angular sees that the ngIf directive is being applied to the div element, but since there is no * or <template> tag, it is unable to locate a template, hence the error.


If you get this error with Angular v5:

Error: StaticInjectorError[TemplateRef]:
  StaticInjectorError[TemplateRef]:
    NullInjectorError: No provider for TemplateRef!

You may have <template>...</template> in one or more of your component templates. Change/update the tag to <ng-template>...</ng-template>.

How to use tick / checkmark symbol (?) instead of bullets in unordered list?

You can use a pseudo-element to insert that character before each list item:

_x000D_
_x000D_
ul {_x000D_
  list-style: none;_x000D_
}_x000D_
_x000D_
ul li:before {_x000D_
  content: '?';_x000D_
}
_x000D_
<ul>_x000D_
  <li>this is my text</li>_x000D_
  <li>this is my text</li>_x000D_
  <li>this is my text</li>_x000D_
  <li>this is my text</li>_x000D_
  <li>this is my text</li>_x000D_
</ul>
_x000D_
_x000D_
_x000D_

NullPointerException: Attempt to invoke virtual method 'int java.util.ArrayList.size()' on a null object reference

This issue is due to ArrayList variable not being instantiated. Need to declare "recordings" variable like following, that should solve the issue;

ArrayList<String> recordings = new ArrayList<String>();

this calls default constructor and assigns empty string to the recordings variable so that it is not null anymore.

Add swipe to delete UITableViewCell

    import UIKit

    class ViewController: UIViewController ,UITableViewDelegate,UITableViewDataSource
    {
      var items: String[] = ["We", "Heart", "Swift","omnamay shivay","om namay bhagwate vasudeva nama"]
        var cell : UITableViewCell
}




    @IBOutlet var tableview:UITableView

    override func viewDidLoad() {
        super.viewDidLoad()
        // Do any additional setup after loading the view, typically from a nib.


    }

    override func didReceiveMemoryWarning() {
        super.didReceiveMemoryWarning()
        // Dispose of any resources that can be recreated.
    }




    func tableView(tableView: UITableView!, numberOfRowsInSection section: Int) -> Int {
        return self.items.count;
    }


    func tableView(tableView: UITableView!, cellForRowAtIndexPath indexPath: NSIndexPath!) -> UITableViewCell! {

        var cell = tableView.dequeueReusableCellWithIdentifier("CELL") as? UITableViewCell

        if !cell {
            cell = UITableViewCell(style: UITableViewCellStyle.Value1, reuseIdentifier: "CELL")}



        cell!.textLabel.text = self.items[indexPath.row]

        return cell
        }
    func tableView(tableView: UITableView!, canEditRowAtIndexPath indexPath: NSIndexPath!) -> Bool {
        return true
    }

    func tableView(tableView: UITableView!, commitEditingStyle editingStyle: UITableViewCellEditingStyle, forRowAtIndexPath indexPath: NSIndexPath!) {
        if (editingStyle == UITableViewCellEditingStyle.Delete) {
            // handle delete (by removing the data from your array and updating the tableview)


            if let tv=tableView
            {



             items.removeAtIndex(indexPath!.row)
                tv.deleteRowsAtIndexPaths([indexPath], withRowAnimation: .Fade)



        }
    }
}


}

how to check for null with a ng-if values in a view with angularjs?

See the correct way with your example:

<div ng-if="!test.view">1</div>
<div ng-if="!!test.view">2</div>

Regards, Nicholls

Radio Buttons ng-checked with ng-model

I think you should only use ng-model and should work well for you, here is the link to the official documentation of angular https://docs.angularjs.org/api/ng/input/input%5Bradio%5D

The code from the example should not be difficult to adapt to your specific situation:

<script>
   function Ctrl($scope) {
      $scope.color = 'blue';
      $scope.specialValue = {
         "id": "12345",
         "value": "green"
      };
   }
</script>
<form name="myForm" ng-controller="Ctrl">
   <input type="radio" ng-model="color" value="red">  Red <br/>
   <input type="radio" ng-model="color" ng-value="specialValue"> Green <br/>
   <input type="radio" ng-model="color" value="blue"> Blue <br/>
   <tt>color = {{color | json}}</tt><br/>
</form>

How to draw a checkmark / tick using CSS?

Also, using the awesome font, you can use the following tag. Simple and beautiful

With the possibility of changing the size and color and other features in CSS

See result here

MySQL Daemon Failed to Start - centos 6

Reference here 2.10.2.1 Troubleshooting Problems Starting the MySQL Server.

1.Find the data directory ,it was configured in my.cnf.

[mysqld]
datadir=/var/lib/mysql

2. Check the err file,it log the error message about why mysql server start failed. the name of err file is related with your hostname.

cd /var/lib/mysql
ll
tail (hostname).err

3.If you find some messages like :

InnoDB: Error: log file ./ib_logfile0 is of different size 0 33554432 bytes
InnoDB: than specified in the .cnf file 0 5242880 bytes!
170513 14:25:22 [ERROR] Plugin 'InnoDB' init function returned error.
170513 14:25:22 [ERROR] Plugin 'InnoDB' registration as a STORAGE ENGINE failed.
170513 14:25:22 [ERROR] Unknown/unsupported storage engine: InnoDB
170513 14:25:22 [ERROR] Aborting

then

delete ib_logfile0 and ib_logfile1

, then,

/etc/init.d/mysqld start

C++ callback using class member

If you have callbacks with different parameters you can use templates as follows:
// compile with: g++ -std=c++11 myTemplatedCPPcallbacks.cpp -o myTemplatedCPPcallbacksApp

#include <functional>     // c++11

#include <iostream>        // due to: cout


using std::cout;
using std::endl;

class MyClass
{
    public:
        MyClass();
        static void Callback(MyClass* instance, int x);
    private:
        int private_x;
};

class OtherClass
{
    public:
        OtherClass();
        static void Callback(OtherClass* instance, std::string str);
    private:
        std::string private_str;
};

class EventHandler
{

    public:
        template<typename T, class T2>
        void addHandler(T* owner, T2 arg2)
        {
            cout << "\nHandler added..." << endl;
            //Let's pretend an event just occured
            owner->Callback(owner, arg2);
         }   

};

MyClass::MyClass()
{
    EventHandler* handler;
    private_x = 4;
    handler->addHandler(this, private_x);
}

OtherClass::OtherClass()
{
    EventHandler* handler;
    private_str = "moh ";
    handler->addHandler(this, private_str );
}

void MyClass::Callback(MyClass* instance, int x)
{
    cout << " MyClass::Callback(MyClass* instance, int x) ==> " 
         << 6 + x + instance->private_x << endl;
}

void OtherClass::Callback(OtherClass* instance, std::string private_str)
{
    cout << " OtherClass::Callback(OtherClass* instance, std::string private_str) ==> " 
         << " Hello " << instance->private_str << endl;
}

int main(int argc, char** argv)
{
    EventHandler* handler;
    handler = new EventHandler();
    MyClass* myClass = new MyClass();
    OtherClass* myOtherClass = new OtherClass();
}

Conditionally change img src based on model data

For angular 4 I have used

<img [src]="data.pic ? data.pic : 'assets/images/no-image.png' " alt="Image" title="Image">

It works for me , I hope it may use to other's also for Angular 4-5. :)

Regex to Match Symbols: !$%^&*()_+|~-=`{}[]:";'<>?,./

The regular expression for this is really simple. Just use a character class. The hyphen is a special character in character classes, so it needs to be first:

/[-!$%^&*()_+|~=`{}\[\]:";'<>?,.\/]/

You also need to escape the other regular expression metacharacters.

Edit: The hyphen is special because it can be used to represent a range of characters. This same character class can be simplified with ranges to this:

/[$-/:-?{-~!"^_`\[\]]/

There are three ranges. '$' to '/', ':' to '?', and '{' to '~'. the last string of characters can't be represented more simply with a range: !"^_`[].

Use an ACSII table to find ranges for character classes.

jQuery checkbox change and click event

Late answer, but you can also use on("change")

_x000D_
_x000D_
$('#check').on('change', function() {
     var checked = this.checked
    $('span').html(checked.toString())
});
_x000D_
<script src="https://cdnjs.cloudflare.com/ajax/libs/jquery/3.3.1/jquery.min.js"></script>
<input type="checkbox" id="check"> <span>Check me!</span>
_x000D_
_x000D_
_x000D_

Programmatically set left drawable in a TextView

Using Kotlin:

You can create an extension function or just use setCompoundDrawablesWithIntrinsicBounds directly.

fun TextView.leftDrawable(@DrawableRes id: Int = 0) {
   this.setCompoundDrawablesWithIntrinsicBounds(id, 0, 0, 0)
}

If you need to resize the drawable, you can use this extension function.

textView.leftDrawable(R.drawable.my_icon, R.dimen.icon_size)

fun TextView.leftDrawable(@DrawableRes id: Int = 0, @DimenRes sizeRes: Int) {
    val drawable = ContextCompat.getDrawable(context, id)
    val size = resources.getDimensionPixelSize(sizeRes)
    drawable?.setBounds(0, 0, size, size)
    this.setCompoundDrawables(drawable, null, null, null)
}

To get really fancy, create a wrapper that allows size and/or color modification.

textView.leftDrawable(R.drawable.my_icon, colorRes = R.color.white)

fun TextView.leftDrawable(@DrawableRes id: Int = 0, @DimenRes sizeRes: Int = 0, @ColorInt color: Int = 0, @ColorRes colorRes: Int = 0) {
    val drawable = drawable(id)
    if (sizeRes != 0) {
        val size = resources.getDimensionPixelSize(sizeRes)
        drawable?.setBounds(0, 0, size, size)
    }
    if (color != 0) {
        drawable?.setColorFilter(color, PorterDuff.Mode.SRC_ATOP)
    } else if (colorRes != 0) {
        val colorInt = ContextCompat.getColor(context, colorRes)
        drawable?.setColorFilter(colorInt, PorterDuff.Mode.SRC_ATOP)
    }
    this.setCompoundDrawables(drawable, null, null, null)
}

How to set space between listView Items in Android

I realize that an answer was already been selected, but I just wanted to share what ended up working for me when I ran into this issue.

I had a listView where each entry in the listView was defined by its own layout, similar to what Sammy posted in his question. I tried the suggested approach of changing the divider height, but that did not end up looking all too pretty, even with an invisible divider. After some experimentation, I simply added an android:paddingBottom="5dip" to the last TextView layout element in the XML file that defines individual listView entries.

This ended up giving me exactly what I was trying to achieve via the use of android:layout_marginBottom. I found this solution to produce a more aesthetically pleasing result than trying to increase the divider height.

In HTML I can make a checkmark with &#x2713; . Is there a corresponding X-mark?

Personally, I like to use named entities when they are available, because they make my HTML more readable. Because of that, I like to use &check; for ✓ and &cross; for ✗. If you're not sure whether a named entity exists for the character you want, try the &what search site. It includes the name for each entity, if there is one.

As mentioned in the comments, &check; and &cross; are not supported in HTML4, so you may be better off using the more cryptic &#x2713; and &#x2717; if you want to target the most browsers. The most definitive references I could find were on the W3C site: HTML4 and HTML5.

Tick symbol in HTML/XHTML

you could use ⊕ or ⊗

Python, Unicode, and the Windows console

The cause of your problem is NOT the Win console not willing to accept Unicode (as it does this since I guess Win2k by default). It is the default system encoding. Try this code and see what it gives you:

import sys
sys.getdefaultencoding()

if it says ascii, there's your cause ;-) You have to create a file called sitecustomize.py and put it under python path (I put it under /usr/lib/python2.5/site-packages, but that is differen on Win - it is c:\python\lib\site-packages or something), with the following contents:

import sys
sys.setdefaultencoding('utf-8')

and perhaps you might want to specify the encoding in your files as well:

# -*- coding: UTF-8 -*-
import sys,time

Edit: more info can be found in excellent the Dive into Python book

Go to first line in a file in vim?

If you are using gvim, you could just hit Ctrl + Home to go the first line. Similarly, Ctrl + End goes to the last line.

Use PHP to create, edit and delete crontab jobs?

Nice...
Try this to remove an specific cron job (tested).

<?php $output = shell_exec('crontab -l'); ?>
<?php $cron_file = "/tmp/crontab.txt"; ?>

<!-- Execute script when form is submitted -->
<?php if(isset($_POST['add_cron'])) { ?>

<!-- Add new cron job -->
<?php if(!empty($_POST['add_cron'])) { ?>
<?php file_put_contents($cron_file, $output.$_POST['add_cron'].PHP_EOL); ?>
<?php } ?>

<!-- Remove cron job -->
<?php if(!empty($_POST['remove_cron'])) { ?>
<?php $remove_cron = str_replace($_POST['remove_cron']."\n", "", $output); ?>
<?php file_put_contents($cron_file, $remove_cron.PHP_EOL); ?>
<?php } ?>

<!-- Remove all cron jobs -->
<?php if(isset($_POST['remove_all_cron'])) { ?>
<?php echo exec("crontab -r"); ?>
<?php } else { ?>
<?php echo exec("crontab $cron_file"); ?>
<?php } ?>

<!-- Reload page to get updated cron jobs -->
<?php $uri = $_SERVER['REQUEST_URI']; ?>
<?php header("Location: $uri"); ?>
<?php exit; ?>
<?php } ?>

<b>Current Cron Jobs:</b><br>
<?php echo nl2br($output); ?>

<h2>Add or Remove Cron Job</h2>
<form method="post" action="<?php $_SERVER['REQUEST_URI']; ?>">
<b>Add New Cron Job:</b><br>
<input type="text" name="add_cron" size="100" placeholder="e.g.: * * * * * /usr/local/bin/php -q /home/username/public_html/my_cron.php"><br>
<b>Remove Cron Job:</b><br>
<input type="text" name="remove_cron" size="100" placeholder="e.g.: * * * * * /usr/local/bin/php -q /home/username/public_html/my_cron.php"><br>
<input type="checkbox" name="remove_all_cron" value="1"> Remove all cron jobs?<br>
<input type="submit"><br>
</form>

What does upstream mean in nginx?

If we have a single server we can directly include it in the proxy_pass. But in case if we have many servers we use upstream to maintain the servers. Nginx will load-balance based on the incoming traffic.

what does this mean ? image/png;base64?

That data:image/png;base64 URL is cool, I’ve never run into it before. The long encrypted link is the actual image, i.e. no image call to the server. See RFC 2397 for details.

Side note: I have had trouble getting larger base64 images to render on IE8. I believe IE8 has a 32K limit that can be problematic for larger files. See this other StackOverflow thread for details.

Android Activity without ActionBar

Haha, I have been stuck at that point a while ago as well, so I am glad I can help you out with a solution, that worked for me at least :)

What you want to do is define a new style within values/styles.xml so it looks like this

<resources>
    <style name = "AppTheme" parent = "android:Theme.Holo.Light.DarkActionBar">
        <!-- Customize your theme here. -->
    </style>

    <style name = "NoActionBar" parent = "@android:style/Theme.Holo.Light">
        <item name = "android:windowActionBar">false</item>
        <item name = "android:windowNoTitle">true</item>
    </style>

</resources>

Only the NoActionBar style is intresting for you. At last you have to set is as your application's theme in the AndroidManifest.xml so it looks like this

<application
    android:allowBackup = "true"
    android:icon = "@drawable/ic_launcher"
    android:label = "@string/app_name"
    android:theme = "@style/NoActionBar"   <!--This is the important line-->
    >
    <activity
    [...]

I hope this helps, if not, let me know.

Running multiple AsyncTasks at the same time -- not possible?

It is posible. My android device version is 4.0.4 and android.os.Build.VERSION.SDK_INT is 15

I have 3 spinners

Spinner c_fruit=(Spinner) findViewById(R.id.fruits);
Spinner c_vegetable=(Spinner) findViewById(R.id.vegetables);
Spinner c_beverage=(Spinner) findViewById(R.id.beverages);

And also I have a Async-Tack class.

Here is my spinner loading code

RequestSend reqs_fruit = new RequestSend(this);
reqs_fruit.where="Get_fruit_List";
reqs_fruit.title="Loading fruit";
reqs_fruit.execute();

RequestSend reqs_vegetable = new RequestSend(this);
reqs_vegetable.where="Get_vegetable_List";
reqs_vegetable.title="Loading vegetable";
reqs_vegetable.execute();

RequestSend reqs_beverage = new RequestSend(this);
reqs_beverage.where="Get_beverage_List";
reqs_beverage.title="Loading beverage";
reqs_beverage.execute();

This is working perfectly. One by one my spinners loaded. I didn't user executeOnExecutor.

Here is my Async-task class

public class RequestSend  extends AsyncTask<String, String, String > {

    private ProgressDialog dialog = null;
    public Spinner spin;
    public String where;
    public String title;
    Context con;
    Activity activity;      
    String[] items;

    public RequestSend(Context activityContext) {
        con = activityContext;
        dialog = new ProgressDialog(activityContext);
        this.activity = activityContext;
    }

    @Override
    protected void onPostExecute(String result) {
        try {
            ArrayAdapter<String> adapter = new ArrayAdapter<String> (activity, android.R.layout.simple_spinner_item, items);       
            adapter.setDropDownViewResource(android.R.layout.simple_spinner_dropdown_item);
            spin.setAdapter(adapter);
        } catch (NullPointerException e) {
            Toast.makeText(activity, "Can not load list. Check your connection", Toast.LENGTH_LONG).show();
            e.printStackTrace();
        } catch (Exception e)  {
            Toast.makeText(activity, "Can not load list. Check your connection", Toast.LENGTH_LONG).show();
            e.printStackTrace();
        }
        super.onPostExecute(result);

        if (dialog != null)
            dialog.dismiss();   
    }

    protected void onPreExecute() {
        super.onPreExecute();
        dialog.setTitle(title);
        dialog.setMessage("Wait...");
        dialog.setCancelable(false); 
        dialog.show();
    }

    @Override
    protected String doInBackground(String... Strings) {
        try {
            Send_Request();
            } catch (NullPointerException e) {
                e.printStackTrace();
            } catch (Exception e) {
                e.printStackTrace();
            }
        return null;
    }

    public void Send_Request() throws JSONException {

        try {
            String DataSendingTo = "http://www.example.com/AppRequest/" + where;
            //HttpClient
            HttpClient httpClient = new DefaultHttpClient();
            //Post header
            HttpPost httpPost = new HttpPost(DataSendingTo);
            //Adding data
            List<NameValuePair> nameValuePairs = new ArrayList<NameValuePair>(2);

            nameValuePairs.add(new BasicNameValuePair("authorized","001"));

            httpPost.setEntity(new UrlEncodedFormEntity(nameValuePairs));
            // execute HTTP post request
            HttpResponse response = httpClient.execute(httpPost);

            BufferedReader reader;
            try {
                reader = new BufferedReader(new InputStreamReader(response.getEntity().getContent()));
                StringBuilder builder = new StringBuilder();
                String line = null;
                while ((line = reader.readLine()) != null) {
                    builder.append(line) ;
                }

                JSONTokener tokener = new JSONTokener(builder.toString());
                JSONArray finalResult = new JSONArray(tokener);
                items = new String[finalResult.length()]; 
                // looping through All details and store in public String array
                for(int i = 0; i < finalResult.length(); i++) {
                    JSONObject c = finalResult.getJSONObject(i);
                    items[i]=c.getString("data_name");
                }

            } catch (ClientProtocolException e) {
                e.printStackTrace();
            } catch (IOException e) {
                e.printStackTrace();
            }

        } catch (ClientProtocolException e) {
            e.printStackTrace();
        } catch (IOException e) {
            e.printStackTrace();
        }
    }
}

Use jQuery to change an HTML tag?

Idea is to wrap the element & unwrap the contents:

function renameElement($element,newElement){

    $element.wrap("<"+newElement+">");
    $newElement = $element.parent();

    //Copying Attributes
    $.each($element.prop('attributes'), function() {
        $newElement.attr(this.name,this.value);
    });

    $element.contents().unwrap();       

    return $newElement;
}

Sample usage:

renameElement($('p'),'h5');

Demo

How to prevent IFRAME from redirecting top-level window

By doing so you'd be able to control any action of the framed page, which you cannot. Same-domain origin policy applies.

Bogus foreign key constraint fail

On demand, now as an answer...

When using MySQL Query Browser or phpMyAdmin, it appears that a new connection is opened for each query (bugs.mysql.com/bug.php?id=8280), making it neccessary to write all the drop statements in one query, eg.

SET FOREIGN_KEY_CHECKS=0; 
DROP TABLE my_first_table_to_drop; 
DROP TABLE my_second_table_to_drop; 
SET FOREIGN_KEY_CHECKS=1; 

Where the SET FOREIGN_KEY_CHECKS=1 serves as an extra security measure...

Undefined reference to main - collect2: ld returned 1 exit status

Perhaps your main function has been commented out because of e.g. preprocessing. To learn what preprocessing is doing, try gcc -C -E es3.c > es3.i then look with an editor into the generated file es3.i (and search main inside it).

First, you should always (since you are a newbie) compile with

  gcc -Wall -g -c es3.c
  gcc -Wall -g es3.o -o es3

The -Wall flag is extremely important, and you should always use it. It tells the compiler to give you (almost) all warnings. And you should always listen to the warnings, i.e. correct your source code file es3.C till you got no more warnings.

The -g flag is important also, because it asks gcc to put debugging information in the object file and the executable. Then you are able to use a debugger (like gdb) to debug your program.

To get the list of symbols in an object file or an executable, you can use nm.

Of course, I'm assuming you use a GNU/Linux system (and I invite you to use GNU/Linux if you don't use it already).

How to move certain commits to be based on another branch in git?

The simplest thing you can do is cherry picking a range. It does the same as the rebase --onto but is easier for the eyes :)

git cherry-pick quickfix1..quickfix2

Why does sudo change the PATH?

I think it is in fact desirable to have sudo reset the PATH: otherwise an attacker having compromised your user account could put backdoored versions of all kinds of tools on your users' PATH, and they would be executed when using sudo.

(of course having sudo reset the PATH is not a complete solution to these kinds of problems, but it helps)

This is indeed what happens when you use

Defaults env_reset

in /etc/sudoers without using exempt_group or env_keep.

This is also convenient because you can add directories that are only useful for root (such as /sbin and /usr/sbin) to the sudo path without adding them to your users' paths. To specify the path to be used by sudo:

Defaults secure_path="/bin:/usr/bin:/usr/local/bin:/sbin:/usr/sbin"

SELECT * FROM X WHERE id IN (...) with Dapper ORM

Here is possibly the fastest way to query a large number of rows with Dapper using a list of IDs. I promise you this is faster than almost any other way you can think of (with the possible exception of using a TVP as given in another answer, and which I haven't tested, but I suspect may be slower because you still have to populate the TVP). It is planets faster than Dapper using IN syntax and universes faster than Entity Framework row by row. And it is even continents faster than passing in a list of VALUES or UNION ALL SELECT items. It can easily be extended to use a multi-column key, just add the extra columns to the DataTable, the temp table, and the join conditions.

public IReadOnlyCollection<Item> GetItemsByItemIds(IEnumerable<int> items) {
   var itemList = new HashSet(items);
   if (itemList.Count == 0) { return Enumerable.Empty<Item>().ToList().AsReadOnly(); }

   var itemDataTable = new DataTable();
   itemDataTable.Columns.Add("ItemId", typeof(int));
   itemList.ForEach(itemid => itemDataTable.Rows.Add(itemid));

   using (SqlConnection conn = GetConnection()) // however you get a connection
   using (var transaction = conn.BeginTransaction()) {
      conn.Execute(
         "CREATE TABLE #Items (ItemId int NOT NULL PRIMARY KEY CLUSTERED);",
         transaction: transaction
      );

      new SqlBulkCopy(conn, SqlBulkCopyOptions.Default, transaction) {
         DestinationTableName = "#Items",
         BulkCopyTimeout = 3600 // ridiculously large
      }
         .WriteToServer(itemDataTable);
      var result = conn
         .Query<Item>(@"
            SELECT i.ItemId, i.ItemName
            FROM #Items x INNER JOIN dbo.Items i ON x.ItemId = i.ItemId
            DROP TABLE #Items;",
            transaction: transaction,
            commandTimeout: 3600
         )
         .ToList()
         .AsReadOnly();
      transaction.Rollback(); // Or commit if you like
      return result;
   }
}

Be aware that you need to learn a little bit about Bulk Inserts. There are options about firing triggers (the default is no), respecting constraints, locking the table, allowing concurrent inserts, and so on.

add string to String array

Since Java arrays hold a fixed number of values, you need to create a new array with a length of 5 in this case. A better solution would be to use an ArrayList and simply add strings to the array.

Example:

ArrayList<String> scripts = new ArrayList<String>();
scripts.add("test3");
scripts.add("test4");
scripts.add("test5");

// Then later you can add more Strings to the ArrayList
scripts.add("test1");
scripts.add("test2");

NGinx Default public www location?

If installing on Ubuntu using apt-get, try /usr/share/nginx/www.

EDIT:

On more recent versions the path has changed to: /usr/share/nginx/html

2019 EDIT:

Might try in /var/www/html/index.nginx-debian.html too.

How to show data in a table by using psql command line interface?

Newer versions: (from 8.4 - mentioned in release notes)

TABLE mytablename;

Longer but works on all versions:

SELECT * FROM mytablename;

You may wish to use \x first if it's a wide table, for readability.

For long data:

SELECT * FROM mytable LIMIT 10;

or similar.

For wide data (big rows), in the psql command line client, it's useful to use \x to show the rows in key/value form instead of tabulated, e.g.

 \x
SELECT * FROM mytable LIMIT 10;

Note that in all cases the semicolon at the end is important.

In OS X Lion, LANG is not set to UTF-8, how to fix it?

I noticed the exact same issue when logging onto servers running Red Hat from an OSX Lion machine.

Try adding or editing the ~/.profile file for it to correctly export your locale settings upon initiating a new session.

export LC_ALL=en_US.UTF-8  
export LANG=en_US.UTF-8

These two lines added to the file should suffice to set the locale [replace en_US for your desired locale, and check beforehand that it is indeed installed on your system (locale -a)].

After that, you can start a new session and check using locale:

$ locale

The following should be the output:

LANG="en_US.UTF-8"  
LC_COLLATE="en_US.UTF-8"  
LC_CTYPE="en_US.UTF-8"  
LC_MESSAGES="en_US.UTF-8"  
LC_MONETARY="en_US.UTF-8"  
LC_NUMERIC="en_US.UTF-8"  
LC_TIME="en_US.UTF-8"  
LC_ALL="en_US.UTF-8"  

jquery fill dropdown with json data

Here is an example of code, that attempts to featch AJAX data from /Ajax/_AjaxGetItemListHelp/ URL. Upon success, it removes all items from dropdown list with id = OfferTransModel_ItemID and then it fills it with new items based on AJAX call's result:

if (productgrpid != 0) {    
    $.ajax({
        type: "POST",
        url: "/Ajax/_AjaxGetItemListHelp/",
        data:{text:"sam",OfferTransModel_ItemGrpid:productgrpid},
        contentType: "application/json",              
        dataType: "json",
        success: function (data) {
            $("#OfferTransModel_ItemID").empty();

            $.each(data, function () {
                $("#OfferTransModel_ItemID").append($("<option>                                                      
                </option>").val(this['ITEMID']).html(this['ITEMDESC']));
            });
        }
    });
}

Returned AJAX result is expected to return data encoded as AJAX array, where each item contains ITEMID and ITEMDESC elements. For example:

{
    {
        "ITEMID":"13",
        "ITEMDESC":"About"
    },
    {
        "ITEMID":"21",
        "ITEMDESC":"Contact"
    }
}

The OfferTransModel_ItemID listbox is populated with above data and its code should look like:

<select id="OfferTransModel_ItemID" name="OfferTransModel[ItemID]">
    <option value="13">About</option>
    <option value="21">Contact</option>
</select>

When user selects About, form submits 13 as value for this field and 21 when user selects Contact and so on.

Fell free to modify above code if your server returns URL in a different format.

Making an array of integers in iOS

I think it's a lot easier to use NSNumbers. This all you need to do:

NSNumber *myNum1 = [NSNumber numberWithInt:myNsIntValue1];
NSNumber *myNum2 = [NSNumber numberWithInt:myNsIntValue2];
.
.
.
NSArray *myArray = [NSArray arrayWithObjects: myNum1, myNum2, ..., nil];

How to get value by key from JObject?

You can also get the value of an item in the jObject like this:

JToken value;
if (json.TryGetValue(key, out value))
{
   DoSomething(value);
}

What is the difference between SQL and MySQL?

SQL stands for Structured Query Language, and is the basis for which all Relational Database Management Systems allow the user to add, remove, update, or select records. Things like MySQ are the actual Management Systems which allow you to store and retrieve your data, whereas SQL is the actual language to do so.

The basic SQL is somewhat universal - Selects usually look the same, Inserts, Updates, Deletes, etc. Once you get beyond the basics, the commands and abilities of your individual Databases vary, and this is where you get people who are Oracle experts, MySQL, SQL Server, etc.

Basically, MySQL is one of many books holding everything, and SQL is how you go about reading that book.

For loop in multidimensional javascript array

A bit too late, but this solution is nice and neat

const arr = [[1,2,3],[4,5,6],[7,8,9,10]]
for (let i of arr) {
  for (let j of i) {
    console.log(j) //Should log numbers from 1 to 10
  }
}

Or in your case:

const arr = [[1,2,3],[4,5,6],[7,8,9]]
for (let [d1, d2, d3] of arr) {
  console.log(`${d1}, ${d2}, ${d3}`) //Should return numbers from 1 to 9
}

Note: for ... of loop is standardised in ES6, so only use this if you have an ES5 Javascript Complier (such as Babel)

Another note: There are alternatives, but they have some subtle differences and behaviours, such as forEach(), for...in, for...of and traditional for(). It depends on your case to decide which one to use. (ES6 also has .map(), .filter(), .find(), .reduce())

Can I do Android Programming in C++, C?

You should look at MoSync too, MoSync gives you standard C/C++, easy-to-use well-documented APIs, and a full-featured Eclipse-based IDE. Its now a open sourced IDE still pretty cool but not maintained anymore.

RadioGroup: How to check programmatically

You may need to declare the radio buttons in the onCreate method of your code and use them.

RadioButton rb1 = (RadioButton) findViewById(R.id.option1);
rb1.setChecked(true);

Can you explain the HttpURLConnection connection process?

String message = URLEncoder.encode("my message", "UTF-8");

try {
    // instantiate the URL object with the target URL of the resource to
    // request
    URL url = new URL("http://www.example.com/comment");

    // instantiate the HttpURLConnection with the URL object - A new
    // connection is opened every time by calling the openConnection
    // method of the protocol handler for this URL.
    // 1. This is the point where the connection is opened.
    HttpURLConnection connection = (HttpURLConnection) url
            .openConnection();
    // set connection output to true
    connection.setDoOutput(true);
    // instead of a GET, we're going to send using method="POST"
    connection.setRequestMethod("POST");

    // instantiate OutputStreamWriter using the output stream, returned
    // from getOutputStream, that writes to this connection.
    // 2. This is the point where you'll know if the connection was
    // successfully established. If an I/O error occurs while creating
    // the output stream, you'll see an IOException.
    OutputStreamWriter writer = new OutputStreamWriter(
            connection.getOutputStream());

    // write data to the connection. This is data that you are sending
    // to the server
    // 3. No. Sending the data is conducted here. We established the
    // connection with getOutputStream
    writer.write("message=" + message);

    // Closes this output stream and releases any system resources
    // associated with this stream. At this point, we've sent all the
    // data. Only the outputStream is closed at this point, not the
    // actual connection
    writer.close();
    // if there is a response code AND that response code is 200 OK, do
    // stuff in the first if block
    if (connection.getResponseCode() == HttpURLConnection.HTTP_OK) {
        // OK

        // otherwise, if any other status code is returned, or no status
        // code is returned, do stuff in the else block
    } else {
        // Server returned HTTP error code.
    }
} catch (MalformedURLException e) {
    // ...
} catch (IOException e) {
    // ...
}

The first 3 answers to your questions are listed as inline comments, beside each method, in the example HTTP POST above.

From getOutputStream:

Returns an output stream that writes to this connection.

Basically, I think you have a good understanding of how this works, so let me just reiterate in layman's terms. getOutputStream basically opens a connection stream, with the intention of writing data to the server. In the above code example "message" could be a comment that we're sending to the server that represents a comment left on a post. When you see getOutputStream, you're opening the connection stream for writing, but you don't actually write any data until you call writer.write("message=" + message);.

From getInputStream():

Returns an input stream that reads from this open connection. A SocketTimeoutException can be thrown when reading from the returned input stream if the read timeout expires before data is available for read.

getInputStream does the opposite. Like getOutputStream, it also opens a connection stream, but the intent is to read data from the server, not write to it. If the connection or stream-opening fails, you'll see a SocketTimeoutException.

How about the getInputStream? Since I'm only able to get the response at getInputStream, then does it mean that I didn't send any request at getOutputStream yet but simply establishes a connection?

Keep in mind that sending a request and sending data are two different operations. When you invoke getOutputStream or getInputStream url.openConnection(), you send a request to the server to establish a connection. There is a handshake that occurs where the server sends back an acknowledgement to you that the connection is established. It is then at that point in time that you're prepared to send or receive data. Thus, you do not need to call getOutputStream to establish a connection open a stream, unless your purpose for making the request is to send data.

In layman's terms, making a getInputStream request is the equivalent of making a phone call to your friend's house to say "Hey, is it okay if I come over and borrow that pair of vice grips?" and your friend establishes the handshake by saying, "Sure! Come and get it". Then, at that point, the connection is made, you walk to your friend's house, knock on the door, request the vice grips, and walk back to your house.

Using a similar example for getOutputStream would involve calling your friend and saying "Hey, I have that money I owe you, can I send it to you"? Your friend, needing money and sick inside that you kept it for so long, says "Sure, come on over you cheap bastard". So you walk to your friend's house and "POST" the money to him. He then kicks you out and you walk back to your house.

Now, continuing with the layman's example, let's look at some Exceptions. If you called your friend and he wasn't home, that could be a 500 error. If you called and got a disconnected number message because your friend is tired of you borrowing money all the time, that's a 404 page not found. If your phone is dead because you didn't pay the bill, that could be an IOException. (NOTE: This section may not be 100% correct. It's intended to give you a general idea of what's happening in layman's terms.)

Question #5:

Yes, you are correct that openConnection simply creates a new connection object but does not establish it. The connection is established when you call either getInputStream or getOutputStream.

openConnection creates a new connection object. From the URL.openConnection javadocs:

A new connection is opened every time by calling the openConnection method of the protocol handler for this URL.

The connection is established when you call openConnection, and the InputStream, OutputStream, or both, are called when you instantiate them.

Question #6:

To measure the overhead, I generally wrap some very simple timing code around the entire connection block, like so:

long start = System.currentTimeMillis();
log.info("Time so far = " + new Long(System.currentTimeMillis() - start) );

// run the above example code here
log.info("Total time to send/receive data = " + new Long(System.currentTimeMillis() - start) );

I'm sure there are more advanced methods for measuring the request time and overhead, but this generally is sufficient for my needs.

For information on closing connections, which you didn't ask about, see In Java when does a URL connection close?.

How can I use jQuery to move a div across the screen

Here i have done complete bins for above query. below is demo link, i think it may help you

Demo: http://codebins.com/bin/4ldqp9b/1

HTML:

<div id="edge">
  <div class="box" style="top:20; background:#f8a2a4;">
  </div>
  <div class="box" style="top:70; background:#a2f8a4;">
  </div>
  <div class="box" style="top:120; background:#5599fd;">
  </div>
</div>
<br/>
<input type="button" id="btnAnimate" name="btnAnimate" value="Animate" />

CSS:

body{
  background:#ffffef;
}
#edge{
  width:500px;
  height:200px;
  border:1px solid #3377af;
  padding:5px;
}

.box{
  position:absolute;
  left:10;
  width:40px;
  height:40px;
  border:1px solid #a82244;
}

JQuery:

$(function() {
    $("#btnAnimate").click(function() {
        var move = "";
        if ($(".box:eq(0)").css('left') == "10px") {
            move = "+=" + ($("#edge").width() - 35);
        } else {
            move = "-=" + ($("#edge").width() - 35);
        }
        $(".box").animate({
            left: move
        }, 500, function() {
            if ($(".box:eq(0)").css('left') == "475px") {
                $(this).css('background', '#afa799');
            } else {
                $(".box:eq(0)").css('background', '#f8a2a4');
                $(".box:eq(1)").css('background', '#a2f8a4');
                $(".box:eq(2)").css('background', '#5599fd');
            }

        });
    });
});

Demo: http://codebins.com/bin/4ldqp9b/1

What are the basic rules and idioms for operator overloading?

The Decision between Member and Non-member

The binary operators = (assignment), [] (array subscription), -> (member access), as well as the n-ary () (function call) operator, must always be implemented as member functions, because the syntax of the language requires them to.

Other operators can be implemented either as members or as non-members. Some of them, however, usually have to be implemented as non-member functions, because their left operand cannot be modified by you. The most prominent of these are the input and output operators << and >>, whose left operands are stream classes from the standard library which you cannot change.

For all operators where you have to choose to either implement them as a member function or a non-member function, use the following rules of thumb to decide:

  1. If it is a unary operator, implement it as a member function.
  2. If a binary operator treats both operands equally (it leaves them unchanged), implement this operator as a non-member function.
  3. If a binary operator does not treat both of its operands equally (usually it will change its left operand), it might be useful to make it a member function of its left operand’s type, if it has to access the operand's private parts.

Of course, as with all rules of thumb, there are exceptions. If you have a type

enum Month {Jan, Feb, ..., Nov, Dec}

and you want to overload the increment and decrement operators for it, you cannot do this as a member functions, since in C++, enum types cannot have member functions. So you have to overload it as a free function. And operator<() for a class template nested within a class template is much easier to write and read when done as a member function inline in the class definition. But these are indeed rare exceptions.

(However, if you make an exception, do not forget the issue of const-ness for the operand that, for member functions, becomes the implicit this argument. If the operator as a non-member function would take its left-most argument as a const reference, the same operator as a member function needs to have a const at the end to make *this a const reference.)


Continue to Common operators to overload.

PHP salt and hash SHA256 for login password

array hash_algos(void)

echo hash('sha384', 'Message to be hashed'.'salt');

Here is a link to reference http://php.net/manual/en/function.hash.php

How to list active connections on PostgreSQL?

Following will give you active connections/ queries in postgres DB-

SELECT 
    pid
    ,datname
    ,usename
    ,application_name
    ,client_hostname
    ,client_port
    ,backend_start
    ,query_start
    ,query
    ,state
FROM pg_stat_activity
WHERE state = 'active';

You may use 'idle' instead of active to get already executed connections/queries.

Adb install failure: INSTALL_CANCELED_BY_USER

For MIUI OS Device

1) Go to Setting

2) Scroll down to Additional Setting

3) You will find Developer option at bottom

4) Turn this on - Install via USB: Toggle On

By turning this on, It is working charm in my MIUI8 device.

Node Express sending image files as API response

There is an api in Express.

res.sendFile

app.get('/report/:chart_id/:user_id', function (req, res) {
    // res.sendFile(filepath);
});

http://expressjs.com/en/api.html#res.sendFile

SSH -L connection successful, but localhost port forwarding not working "channel 3: open failed: connect failed: Connection refused"

This means the remote vm is not listening to current port i solved this by adding the port in the vm server

SQL Server - calculate elapsed time between two datetime stamps in HH:MM:SS format

See if this helps. I can set variables for Elapsed Days, Hours, Minutes, Seconds. You can format this to your liking or include in a user defined function.

Note: Don't use DateDiff(hh,@Date1,@Date2). It is not reliable! It rounds in unpredictable ways

Given two dates... (Sample Dates: two days, three hours, 10 minutes, 30 seconds difference)

declare @Date1 datetime = '2013-03-08 08:00:00'
declare @Date2 datetime = '2013-03-10 11:10:30'
declare @Days decimal
declare @Hours decimal
declare @Minutes decimal
declare @Seconds decimal

select @Days = DATEDIFF(ss,@Date1,@Date2)/60/60/24 --Days
declare @RemainderDate as datetime = @Date2 - @Days
select @Hours = datediff(ss, @Date1, @RemainderDate)/60/60 --Hours
set @RemainderDate = @RemainderDate - (@Hours/24.0)
select @Minutes = datediff(ss, @Date1, @RemainderDate)/60 --Minutes
set @RemainderDate = @RemainderDate - (@Minutes/24.0/60)
select @Seconds = DATEDIFF(SS, @Date1, @RemainderDate)    
select @Days as ElapsedDays, @Hours as ElapsedHours, @Minutes as ElapsedMinutes, @Seconds as ElapsedSeconds

jQuery: How to get the event object in an event handler function without passing it as an argument?

Write your event handler declaration like this:

<a href="#" onclick="myFunc(event,1,2,3)">click</a>

Then your "myFunc()" function can access the event.

The string value of the "onclick" attribute is converted to a function in a way that's almost exactly the same as the browser (internally) calling the Function constructor:

theAnchor.onclick = new Function("event", theOnclickString);

(except in IE). However, because "event" is a global in IE (it's a window attribute), you'll be able to pass it to the function that way in any browser.

Disable text input history

<input type="text" autocomplete="off" />

jQuery: Check if special characters exists in string

You could also use the whitelist method -

var str = $('#Search').val();
var regex = /[^\w\s]/gi;

if(regex.test(str) == true) {
    alert('Your search string contains illegal characters.');
}

The regex in this example is digits, word characters, underscores (\w) and whitespace (\s). The caret (^) indicates that we are to look for everything that is not in our regex, so look for things that are not word characters, underscores, digits and whitespace.

calling javascript function on OnClientClick event of a Submit button

OnClientClick="SomeMethod()" event of that BUTTON, it return by default "true" so after that function it do postback

for solution use

//use this code in BUTTON  ==>   OnClientClick="return SomeMethod();"

//and your function like this
<script type="text/javascript">
  function SomeMethod(){
    // put your code here 
    return false;
  }
</script>

How to solve WAMP and Skype conflict on Windows 7?

  1. open skype
  2. click tools and go to options
  3. click advanced option from left side
  4. click connection
  5. unchecked (Use port 80 and 443 as alternatives for incoming connection)

SimpleDateFormat returns 24-hour date: how to get 12-hour date?

You can try it like this

  Calendar c= Calendar.getInstance();

  SimpleDateFormat sdf= new SimpleDateFormat("dd/MM/yyyy hh:mm:ss a");
  String str=sdf.format(c.getTime());

How to manually set an authenticated user in Spring Security / SpringMVC

I had the same problem as you a while back. I can't remember the details but the following code got things working for me. This code is used within a Spring Webflow flow, hence the RequestContext and ExternalContext classes. But the part that is most relevant to you is the doAutoLogin method.

public String registerUser(UserRegistrationFormBean userRegistrationFormBean,
                           RequestContext requestContext,
                           ExternalContext externalContext) {

    try {
        Locale userLocale = requestContext.getExternalContext().getLocale();
        this.userService.createNewUser(userRegistrationFormBean, userLocale, Constants.SYSTEM_USER_ID);
        String emailAddress = userRegistrationFormBean.getChooseEmailAddressFormBean().getEmailAddress();
        String password = userRegistrationFormBean.getChoosePasswordFormBean().getPassword();
        doAutoLogin(emailAddress, password, (HttpServletRequest) externalContext.getNativeRequest());
        return "success";

    } catch (EmailAddressNotUniqueException e) {
        MessageResolver messageResolvable 
                = new MessageBuilder().error()
                                      .source(UserRegistrationFormBean.PROPERTYNAME_EMAIL_ADDRESS)
                                      .code("userRegistration.emailAddress.not.unique")
                                      .build();
        requestContext.getMessageContext().addMessage(messageResolvable);
        return "error";
    }

}


private void doAutoLogin(String username, String password, HttpServletRequest request) {

    try {
        // Must be called from request filtered by Spring Security, otherwise SecurityContextHolder is not updated
        UsernamePasswordAuthenticationToken token = new UsernamePasswordAuthenticationToken(username, password);
        token.setDetails(new WebAuthenticationDetails(request));
        Authentication authentication = this.authenticationProvider.authenticate(token);
        logger.debug("Logging in with [{}]", authentication.getPrincipal());
        SecurityContextHolder.getContext().setAuthentication(authentication);
    } catch (Exception e) {
        SecurityContextHolder.getContext().setAuthentication(null);
        logger.error("Failure in autoLogin", e);
    }

}

jQuery jump or scroll to certain position, div or target on the page from button onclick

$("html, body").scrollTop($(element).offset().top); // <-- Also integer can be used

What is the difference between smoke testing and sanity testing?

Smoke Testing

  1. Smoke testing is a wide approach where all areas of the software application are tested without getting into too deep

  2. The test cases for smoke testing of the software can be either manual or automated

  3. Smoke testing is done to ensure whether the main functions of the software application are working or not. During smoke testing of the software, we do not go into finer details.

  4. Smoke testing of the software application is done to check whether the build can be accepted for through software testing

  5. This testing is performed by the developers or testers

  6. Smoke testing exercises the entire system from end to end

  7. Smoke testing is like General Health Check Up

  8. Smoke testing is usually documented or scripted

Santy Testing

  1. Sanity software testing is a narrow regression testing with a focus on one or a small set of areas of functionality of the software application.

  2. Sanity test is generally without test scripts or test cases.

  3. Sanity testing is a cursory software testing type. It is done whenever a quick round of software testing can prove that the software application is functioning according to business / functional requirements.

  4. Sanity testing of the software is to ensure whether the requirements are met or not.

  5. Sanity testing is usually performed by testers

  6. Sanity testing exercises only the particular component of the entire system

  7. Sanity Testing is like specialized health check up

  8. Sanity testing is usually not documented and is unscripted

For more visit Link

Node.js server that accepts POST requests

The following code shows how to read values from an HTML form. As @pimvdb said you need to use the request.on('data'...) to capture the contents of the body.

const http = require('http')

const server = http.createServer(function(request, response) {
  console.dir(request.param)

  if (request.method == 'POST') {
    console.log('POST')
    var body = ''
    request.on('data', function(data) {
      body += data
      console.log('Partial body: ' + body)
    })
    request.on('end', function() {
      console.log('Body: ' + body)
      response.writeHead(200, {'Content-Type': 'text/html'})
      response.end('post received')
    })
  } else {
    console.log('GET')
    var html = `
            <html>
                <body>
                    <form method="post" action="http://localhost:3000">Name: 
                        <input type="text" name="name" />
                        <input type="submit" value="Submit" />
                    </form>
                </body>
            </html>`
    response.writeHead(200, {'Content-Type': 'text/html'})
    response.end(html)
  }
})

const port = 3000
const host = '127.0.0.1'
server.listen(port, host)
console.log(`Listening at http://${host}:${port}`)


If you use something like Express.js and Bodyparser then it would look like this since Express will handle the request.body concatenation

var express = require('express')
var fs = require('fs')
var app = express()

app.use(express.bodyParser())

app.get('/', function(request, response) {
  console.log('GET /')
  var html = `
    <html>
        <body>
            <form method="post" action="http://localhost:3000">Name: 
                <input type="text" name="name" />
                <input type="submit" value="Submit" />
            </form>
        </body>
    </html>`
  response.writeHead(200, {'Content-Type': 'text/html'})
  response.end(html)
})

app.post('/', function(request, response) {
  console.log('POST /')
  console.dir(request.body)
  response.writeHead(200, {'Content-Type': 'text/html'})
  response.end('thanks')
})

port = 3000
app.listen(port)
console.log(`Listening at http://localhost:${port}`)

Convert character to ASCII numeric value in java

Instead of this:

String char = name.substring(0,1); //char="a"

You should use the charAt() method.

char c = name.charAt(0); // c='a'
int ascii = (int)c;

Resizable table columns with jQuery

I've done my own jquery ui widget, just thinking if it's good enough.

https://github.com/OnekO/colresizable

AngularJS: How to make angular load script inside ng-include?

I tried using Google reCAPTCHA explicitly. Here is the example:

// put somewhere in your index.html
<script type="text/javascript">
var onloadCallback = function() {
  grecaptcha.render('your-recaptcha-element', {
    'sitekey' : '6Ldcfv8SAAAAAB1DwJTM6T7qcJhVqhqtss_HzS3z'
  });
};

//link function of Angularjs directive
link: function (scope, element, attrs) {
  ...
  var domElem = '<script src="https://www.google.com/recaptcha/api.js?onload=onloadCallback&render=explicit" async defer></script>';
  $('#your-recaptcha-element').append($compile(domElem)(scope));
}

Necessary to add link tag for favicon.ico?

To choose a different location or file type (e.g. PNG or SVG) for the favicon:
One reason can be that you want to have the icon in a specific location, perhaps in the images folder or something alike. For example:

<link rel="icon" href="_/img/favicon.png">

This diferent location may even be a CDN, just like SO seems to do with <link rel="shortcut icon" href="http://cdn.sstatic.net/stackoverflow/img/favicon.ico">.

To learn more about using other file types like PNG check out this question.

For cache busting purposes:
Add a query string to the path for cache-busting purposes:

<link rel="icon" href="/favicon.ico?v=1.1"> 

Favicons are very heavily cached and this a great way to ensure a refresh.


Footnote about default location:
As far as the first bit of the question: all modern browsers would detect a favicon at the default location, so that's not a reason to use a link for it.


Footnote about rel="icon":
As indicated by @Semanino's answer, using rel="shortcut icon" is an old technique which was required by older versions of Internet Explorer, but in most cases can be replaced by the more correct rel="icon" instruction. The article @Semanino based this on properly links to the appropriate spec which shows a rel value of shortcut isn't a valid option.

Why do I get permission denied when I try use "make" to install something?

On many source packages (e.g. for most GNU software), the building system may know about the DESTDIR make variable, so you can often do:

 make install DESTDIR=/tmp/myinst/
 sudo cp -va /tmp/myinst/ /

The advantage of this approach is that make install don't need to run as root, so you cannot end up with files compiled as root (or root-owned files in your build tree).

Add two textbox values and display the sum in a third textbox automatically

try this

  function sum() {
       var txtFirstNumberValue = document.getElementById('txt1').value;
       var txtSecondNumberValue = document.getElementById('txt2').value;
       if (txtFirstNumberValue == "")
           txtFirstNumberValue = 0;
       if (txtSecondNumberValue == "")
           txtSecondNumberValue = 0;

       var result = parseInt(txtFirstNumberValue) + parseInt(txtSecondNumberValue);
       if (!isNaN(result)) {
           document.getElementById('txt3').value = result;
       }
   }

How can I extract audio from video with ffmpeg?

Use -b:a instead of -ab as -ab is outdated now, also make sure your input file path is correct.

To extract audio from a video I have used below command and its working fine.

String[] complexCommand = {"-y", "-i", inputFileAbsolutePath, "-vn", "-ar", "44100", "-ac", "2", "-b:a", "256k", "-f", "mp3", outputFileAbsolutePath};

Here,

  • -y - Overwrite output files without asking.
  • -i - FFmpeg reads from an arbitrary number of input “files” specified by the -i option
  • -vn - Disable video recording
  • -ar - sets the sampling rate for audio streams if encoded
  • -ac - Set the number of audio channels.
  • -b:a - Set the audio bitrate
  • -f - format

Check out this for my complete sample FFmpeg android project on GitHub.

MySQL Cannot Add Foreign Key Constraint

I had a similar error in creating foreign key in a Many to Many table where the primary key consisted of 2 foreign keys and another normal column. I fixed the issue by correcting the referenced table name i.e. company, as shown in the corrected code below:

create table company_life_cycle__history -- (M-M)
(
company_life_cycle_id tinyint unsigned not null,
Foreign Key (company_life_cycle_id) references company_life_cycle(id) ON DELETE    CASCADE ON UPDATE CASCADE,
company_id MEDIUMINT unsigned not null,
Foreign Key (company_id) references company(id) ON DELETE CASCADE ON UPDATE CASCADE,
activity_on date NOT NULL,
PRIMARY KEY pk_company_life_cycle_history (company_life_cycle_id, company_id,activity_on),
created_on datetime DEFAULT NULL,
updated_on datetime DEFAULT NULL,
created_by varchar(50) DEFAULT NULL,
updated_by varchar(50) DEFAULT NULL
);

PHP preg_match - only allow alphanumeric strings and - _ characters

Here is one equivalent of the accepted answer for the UTF-8 world.

if (!preg_match('/^[\p{L}\p{N}_-]+$/u', $string)){
  //Disallowed Character In $string
}

Explanation:

  • [] => character class definition
  • p{L} => matches any kind of letter character from any language
  • p{N} => matches any kind of numeric character
  • _- => matches underscore and hyphen
  • + => Quantifier — Matches between one to unlimited times (greedy)
  • /u => Unicode modifier. Pattern strings are treated as UTF-16. Also causes escape sequences to match unicode characters

Note, that if the hyphen is the last character in the class definition it does not need to be escaped. If the dash appears elsewhere in the class definition it needs to be escaped, as it will be seen as a range character rather then a hyphen.

How to drop rows of Pandas DataFrame whose value in a certain column is NaN

In datasets having large number of columns its even better to see how many columns contain null values and how many don't.

print("No. of columns containing null values")
print(len(df.columns[df.isna().any()]))

print("No. of columns not containing null values")
print(len(df.columns[df.notna().all()]))

print("Total no. of columns in the dataframe")
print(len(df.columns))

For example in my dataframe it contained 82 columns, of which 19 contained at least one null value.

Further you can also automatically remove cols and rows depending on which has more null values
Here is the code which does this intelligently:

df = df.drop(df.columns[df.isna().sum()>len(df.columns)],axis = 1)
df = df.dropna(axis = 0).reset_index(drop=True)

Note: Above code removes all of your null values. If you want null values, process them before.

Cannot read property length of undefined

The id of the input seems is not WallSearch. Maybe you're confusing that name and id. They are two different properties. name is used to define the name by which the value is posted, while id is the unique identification of the element inside the DOM.

Other possibility is that you have two elements with the same id. The browser will pick any of these (probably the last, maybe the first) and return an element that doesn't support the value property.

Scope 'session' is not active for the current thread; IllegalStateException: No thread-bound request found

As per documentation:

If you are accessing scoped beans within Spring Web MVC, i.e. within a request that is processed by the Spring DispatcherServlet, or DispatcherPortlet, then no special setup is necessary: DispatcherServlet and DispatcherPortlet already expose all relevant state.

If you are runnning outside of Spring MVC ( Not processed by DispatchServlet) you have to use the RequestContextListener Not just ContextLoaderListener .

Add the following in your web.xml

   <listener>
            <listener-class>
                    org.springframework.web.context.request.RequestContextListener 
            </listener-class>
    </listener>        

That will provide session to Spring in order to maintain the beans in that scope

Update : As per other answers , the @Controller only sensible when you are with in Spring MVC Context, So the @Controller is not serving actual purpose in your code. Still you can inject your beans into any where with session scope / request scope ( you don't need Spring MVC / Controller to just inject beans in particular scope) .

Update : RequestContextListener exposes the request to the current Thread only.
You have autowired ReportBuilder in two places

1. ReportPage - You can see Spring injected the Report builder properly here, because we are still in Same web Thread. i did changed the order of your code to make sure the ReportBuilder injected in ReportPage like this.

log.info("ReportBuilder name: {}", reportBuilder.getName());
reportController.getReportData();

i knew the log should go after as per your logic , just for debug purpose i added .


2. UselessTasklet - We got exception , here because this is different thread created by Spring Batch , where the Request is not exposed by RequestContextListener.


You should have different logic to create and inject ReportBuilder instance to Spring Batch ( May Spring Batch Parameters and using Future<ReportBuilder> you can return for future reference)

How do I use a regex in a shell script?

the problem is you're trying to use regex features not supported by grep. namely, your \d won't work. use this instead:

REGEX_DATE="^[[:digit:]]{2}[-/][[:digit:]]{2}[-/][[:digit:]]{4}$"
echo "$1" | grep -qE "${REGEX_DATE}"
echo $?

you need the -E flag to get ERE in order to use {#} style.

Name [jdbc/mydb] is not bound in this Context

For those who use Tomcat with Bitronix, this will fix the problem:

The error indicates that no handler could be found for your datasource 'jdbc/mydb', so you'll need to make sure your tomcat server refers to your bitronix configuration files as needed.

In case you're using btm-config.properties and resources.properties files to configure the datasource, specify these two JVM arguments in tomcat:

(if you already used them, make sure your references are correct):

  • btm.root
  • bitronix.tm.configuration

e.g.

-Dbtm.root="C:\Program Files\Apache Software Foundation\Tomcat 7.0.59" 
-Dbitronix.tm.configuration="C:\Program Files\Apache Software Foundation\Tomcat 7.0.59\conf\btm-config.properties" 

Now, restart your server and check the log.

How to make fixed header table inside scrollable div?

I needed the same and this solution worked the most simple and straightforward way:

http://www.farinspace.com/jquery-scrollable-table-plugin/

I just give an id to the table I want to scroll and put one line in Javascript. That's it!

By the way, first I also thought I want to use a scrollable div, but it is not necessary at all. You can use a div and put it into it, but this solution does just what we need: scrolls the table.

How can I download a file from a URL and save it in Rails?

Here's possibly the simplest way:

IO.copy_stream(URI.open("https://i.pinimg.com/originals/24/17/d6/2417d6b3f3dc236b0b5b80fb00b3a791.png"), 'destination.png')

Using BeautifulSoup to search HTML for string

I have not used BeuatifulSoup but maybe the following can help in some tiny way.

import re
import urllib2
stuff = urllib2.urlopen(your_url_goes_here).read()  # stuff will contain the *entire* page

# Replace the string Python with your desired regex
results = re.findall('(Python)',stuff)

for i in results:
    print i

I'm not suggesting this is a replacement but maybe you can glean some value in the concept until a direct answer comes along.

Date minus 1 year?

Although there are many acceptable answers in response to this question, I don't see any examples of the sub method using the \Datetime object: https://www.php.net/manual/en/datetime.sub.php

So, for reference, you can also use a \DateInterval to modify a \Datetime object:

$date = new \DateTime('2009-01-01');
$date->sub(new \DateInterval('P1Y'));

echo $date->format('Y-m-d');

Which returns:

2008-01-01

For more information about \DateInterval, refer to the documentation: https://www.php.net/manual/en/class.dateinterval.php

Regex to match only letters

JavaScript

If you want to return matched letters:

('Example 123').match(/[A-Z]/gi) // Result: ["E", "x", "a", "m", "p", "l", "e"]

If you want to replace matched letters with stars ('*') for example:

('Example 123').replace(/[A-Z]/gi, '*') //Result: "****** 123"*

Targeting only Firefox with CSS

OK, I've found it. This is probably the cleanest and easiest solution out there and does not rely on JavaScript being turned on.

_x000D_
_x000D_
@-moz-document url-prefix() {
  h1 {
    color: red;
  }
}
_x000D_
<h1>This should be red in FF</h1>
_x000D_
_x000D_
_x000D_

It's based on yet another Mozilla specific CSS extension. There's a whole list for these CSS extensions right here: Mozilla CSS Extensions.

How can I match multiple occurrences with a regex in JavaScript similar to PHP's preg_match_all()?

If you can get away with using map this is a four-line-solution:

_x000D_
_x000D_
var mystring = '1111342=Adam%20Franco&348572=Bob%20Jones';_x000D_
_x000D_
var result = mystring.match(/(&|&amp;)?([^=]+)=([^&]+)/g) || [];_x000D_
result = result.map(function(i) {_x000D_
  return i.match(/(&|&amp;)?([^=]+)=([^&]+)/);_x000D_
});_x000D_
_x000D_
console.log(result);
_x000D_
_x000D_
_x000D_

Ain't pretty, ain't efficient, but at least it is compact. ;)

hidden field in php

Yes, you can access it through GET and POST (trying this simple task would have made you aware of that).

Yes, there are other ways, one of the other "preferred" ways is using sessions. When you would want to use hidden over session is kind of touchy, but any GET / POST data is easily manipulated by the end user. A session is a bit more secure given it is saved to a file on the server and it is much harder for the end user to manipulate without access through the program.

Bootstrap 3 scrollable div for table

A scrolling comes from a box with class pre-scrollable

<div class="pre-scrollable"></div>

There's more examples: http://getbootstrap.com/css/#code-block
Wish it helps.

How can I set the font-family & font-size inside of a div?

Append a semicolon to the following line to fix the issue.

font-family:    Arial, Helvetica, sans-serif;

phonegap open link in browser

With Cordova 5.0 and greater the plugin InAppBrowser is renamed in the Cordova plugin registry, so you should install it using

cordova plugin add cordova-plugin-inappbrowser --save

Then use

_x000D_
_x000D_
<a href="#" onclick="window.open('http://www.kidzout.com', '_system');">www.kidzout.com</a>
_x000D_
_x000D_
_x000D_

Playing m3u8 Files with HTML Video Tag

Might be a little late with the answer but you need to supply the MIME type attribute in the video tag: type="application/x-mpegURL". The video tag I use for a 16:9 stream looks like this.

<video width="352" height="198" controls>
    <source src="playlist.m3u8" type="application/x-mpegURL">
</video>

Python Infinity - Any caveats?

So does C99.

The IEEE 754 floating point representation used by all modern processors has several special bit patterns reserved for positive infinity (sign=0, exp=~0, frac=0), negative infinity (sign=1, exp=~0, frac=0), and many NaN (Not a Number: exp=~0, frac?0).

All you need to worry about: some arithmetic may cause floating point exceptions/traps, but those aren't limited to only these "interesting" constants.

ValueError: object too deep for desired array while using convolution

np.convolve() takes one dimension array. You need to check the input and convert it into 1D.

You can use the np.ravel(), to convert the array to one dimension.

jquery if div id has children

This snippet will determine if the element has children using the :parent selector:

if ($('#myfav').is(':parent')) {
    // do something
}

Note that :parent also considers an element with one or more text nodes to be a parent.

Thus the div elements in <div>some text</div> and <div><span>some text</span></div> will each be considered a parent but <div></div> is not a parent.

Difference between binary semaphore and mutex

On Windows, there are two differences between mutexes and binary semaphores:

  1. A mutex can only be released by the thread which has ownership, i.e. the thread which previously called the Wait function, (or which took ownership when creating it). A semaphore can be released by any thread.

  2. A thread can call a wait function repeatedly on a mutex without blocking. However, if you call a wait function twice on a binary semaphore without releasing the semaphore in between, the thread will block.

Why does the html input with type "number" allow the letter 'e' to be entered in the field?

Because that's exactly how the spec says it should work. The number input can accept floating-point numbers, including negative symbols and the e or E character (where the exponent is the number after the e or E):

A floating-point number consists of the following parts, in exactly the following order:

  1. Optionally, the first character may be a "-" character.
  2. One or more characters in the range "0—9".
  3. Optionally, the following parts, in exactly the following order:
    1. a "." character
    2. one or more characters in the range "0—9"
  4. Optionally, the following parts, in exactly the following order:
    1. a "e" character or "E" character
    2. optionally, a "-" character or "+" character
    3. One or more characters in the range "0—9".

SQL Server: Best way to concatenate multiple columns?

Try using below:

SELECT 
    (RTRIM(LTRIM(col_1))) + (RTRIM(LTRIM(col_2))) AS Col_newname,
    col_1,
    col_2 
FROM 
    s_cols 
WHERE
    col_any_condition = ''
;

How to grant remote access permissions to mysql server for user?

This worked for me. But there was a strange problem that even I tryed first those it didnt affect. I updated phpmyadmin page and got it somehow working.

If you need access to local-xampp-mysql. You can go to xampp-shell -> opening command prompt.

Then mysql -uroot -p --port=3306 or mysql -uroot -p (if there is password set). After that you can grant those acces from mysql shell page (also can work from localhost/phpmyadmin).

Just adding these if somebody find this topic and having beginner problems.

How to increment a JavaScript variable using a button press event

yes, supposing your variable is in the global namespace:

<button onclick="myVar += 1;alert('myVar now equals ' + myVar)">Increment!!</button>

How do I make an auto increment integer field in Django?

class Belly(models.Model):
    belly_id = models.AutoField(primary_key=True)
    belly_name = models.CharField(max_length=50)

******** or *******

class Belly(models.Model):
   belly_name = models.CharField(max_length=50)

The difference is:

The first table has the primary key belly_id (specified as AutoField) and second table has the primary key id (implicitly).

I think no need to use this directly; a primary key field will automatically be added to your model if you don’t specify. Otherwise Check the Django Documentation for AutoField for further details related to AutoField.

How to expire a cookie in 30 minutes using jQuery?

30 minutes is 30 * 60 * 1000 miliseconds. Add that to the current date to specify an expiration date 30 minutes in the future.

 var date = new Date();
 var minutes = 30;
 date.setTime(date.getTime() + (minutes * 60 * 1000));
 $.cookie("example", "foo", { expires: date });

How to use google maps without api key

Now you must have API key. You can generate that in google developer console. Here is LINK to the explanation.

What is the best way to determine a session variable is null or empty in C#?

in this way it can be checked whether such a key is available

if (Session.Dictionary.ContainsKey("Sessionkey"))  --> return Bool

Proper usage of Optional.ifPresent()

Why write complicated code when you could make it simple?

Indeed, if you are absolutely going to use the Optional class, the most simple code is what you have already written ...

if (user.isPresent())
{
    doSomethingWithUser(user.get());
}

This code has the advantages of being

  1. readable
  2. easy to debug (breakpoint)
  3. not tricky

Just because Oracle has added the Optional class in Java 8 doesn't mean that this class must be used in all situation.

Selecting element by data attribute with jQuery

The construction like this: $('[data-XXX=111]') isn't working in Safari 8.0.

If you set data attribute this way: $('div').data('XXX', 111), it only works if you set data attribute directly in DOM like this: $('div').attr('data-XXX', 111).

I think it's because jQuery team optimized garbage collector to prevent memory leaks and heavy operations on DOM rebuilding on each change data attribute.

What is the difference between sed and awk?

sed is a stream editor. It works with streams of characters on a per-line basis. It has a primitive programming language that includes goto-style loops and simple conditionals (in addition to pattern matching and address matching). There are essentially only two "variables": pattern space and hold space. Readability of scripts can be difficult. Mathematical operations are extraordinarily awkward at best.

There are various versions of sed with different levels of support for command line options and language features.

awk is oriented toward delimited fields on a per-line basis. It has much more robust programming constructs including if/else, while, do/while and for (C-style and array iteration). There is complete support for variables and single-dimension associative arrays plus (IMO) kludgey multi-dimension arrays. Mathematical operations resemble those in C. It has printf and functions. The "K" in "AWK" stands for "Kernighan" as in "Kernighan and Ritchie" of the book "C Programming Language" fame (not to forget Aho and Weinberger). One could conceivably write a detector of academic plagiarism using awk.

GNU awk (gawk) has numerous extensions, including true multidimensional arrays in the latest version. There are other variations of awk including mawk and nawk.

Both programs use regular expressions for selecting and processing text.

I would tend to use sed where there are patterns in the text. For example, you could replace all the negative numbers in some text that are in the form "minus-sign followed by a sequence of digits" (e.g. "-231.45") with the "accountant's brackets" form (e.g. "(231.45)") using this (which has room for improvement):

sed 's/-\([0-9.]\+\)/(\1)/g' inputfile

I would use awk when the text looks more like rows and columns or, as awk refers to them "records" and "fields". If I was going to do a similar operation as above, but only on the third field in a simple comma delimited file I might do something like:

awk -F, 'BEGIN {OFS = ","} {gsub("-([0-9.]+)", "(" substr($3, 2) ")", $3); print}' inputfile

Of course those are just very simple examples that don't illustrate the full range of capabilities that each has to offer.

javac: file not found: first.java Usage: javac <options> <source files>

So I had the same problem because I wasn't in the right directory where my file was located. So when I ran javac Example.java (for me) it said it couldn't find it. But I needed to go to where my Example.java file was located. So I used the command cd and right after that the location of the file. That worked for me. Tell me if it helps! Thanks!

libaio.so.1: cannot open shared object file

It looks like a 32/64 bit mismatch. The ldd output shows that mainly libraries from /lib64 are chosen. That would indicate that you have installed a 64 bit version of the Oracle client and have created a 64 bit executable. But libaio.so is probably a 32 bit library and cannot be used for your application.

So you either need a 64 bit version of libaio or you create a 32 bit version of your application.

Change span text?

Replace whatever is in the address bar with this:

javascript:document.getElementById('serverTime').innerHTML='[text here]';

Example.

Python; urllib error: AttributeError: 'bytes' object has no attribute 'read'

I'm not familiar with python 3 yet, but it seems like urllib.request.urlopen().read() returns a byte object rather than string.

You might try to feed it into a StringIO object, or even do a str(response).

How to display alt text for an image in chrome

Here is a simple workaround in jQuery. You can implement it as a user script to apply it to every page you view.

$(function () {
  $('img').live('mouseover', function () {
    var img = $(this); // cache query
    if (img.title) {
      return;
    }
    img.attr('title', img.attr('alt'));
  });
});

I have also implemented this as a Chrome extension called alt. Because it uses jQuery.live, it works with dynamically loaded content, too. I have retired this extension and removed it from the Chrome store.

Error: Could not find or load main class

Project > Clean and then make sure BuildPath > Libraries has the correct Library.

PHP: How can I determine if a variable has a value that is between two distinct constant values?

If you just want to check the value is in Range, use this:

   MIN_VALUE = 1;
   MAX_VALUE = 100;
   $customValue = min(MAX_VALUE,max(MIN_VALUE,$customValue)));

Matching a Forward Slash with a regex

In regular expressions, "/" is a special character which needs to be escaped (AKA flagged by placing a \ before it thus negating any specialized function it might serve).

Here's what you need:

var word = /\/(\w+)/ig; //   /abc Match

Read up on RegEx special characters here: http://www.regular-expressions.info/characters.html

Binding select element to object in Angular

For me its working like this, you can console event.target.value.

<select (change) = "ChangeValue($event)" (ngModel)="opt">   
    <option *ngFor=" let opt of titleArr" [value]="opt"></option>
</select>

Get form data in ReactJS

No need to use refs, you can access using event

function handleSubmit(e) {
    e.preventDefault()
    const {username, password } = e.target.elements
    console.log({username: username.value, password: password.value })
}

<form onSubmit={handleSubmit}>
   <input type="text" id="username"/>
   <input type="text" id="password"/>
   <input type="submit" value="Login" />
</form>

How to chain scope queries with OR instead of AND?

If you're looking to provide a scope (instead of explicitly working on the whole dataset) here's what you should do with Rails 5:

scope :john_or_smith, -> { where(name: "John").or(where(lastname: "Smith")) }

Or:

def self.john_or_smith
  where(name: "John").or(where(lastname: "Smith"))
end

How to Upload Image file in Retrofit 2

Retrofit 2.0 solution

@Multipart
@POST(APIUtils.UPDATE_PROFILE_IMAGE_URL)
public Call<CommonResponse> requestUpdateImage(@PartMap Map<String, RequestBody> map);

and

    Map<String, RequestBody> params = new HashMap<>();

    params.put("newProfilePicture" + "\"; filename=\"" + FilenameUtils.getName(file.getAbsolutePath()), RequestBody.create(MediaType.parse("image/jpg"), file));



 Call<CommonResponse> call = request.requestUpdateImage(params);

you can use
image/jpg image/png image/gif

java.lang.NullPointerException: Attempt to invoke virtual method 'int android.view.View.getImportantForAccessibility()' on a null object reference

it sometimes occurs when we use a custom adapter in any activity of fragment . and we return null object i.e null view so the activity gets confused which view to load , so that is why this exception occurs

shows the position where to change the view

Google Maps: Set Center, Set Center Point and Set more points

Try using this code for v3:

gMap = new google.maps.Map(document.getElementById('map')); 
gMap.setZoom(13);      // This will trigger a zoom_changed on the map
gMap.setCenter(new google.maps.LatLng(37.4419, -122.1419));
gMap.setMapTypeId(google.maps.MapTypeId.ROADMAP);

Uses for the '&quot;' entity in HTML

It is impossible, and unnecessary, to know the motivation for using &quot; in element content, but possible motives include: misunderstanding of HTML rules; use of software that generates such code (probably because its author thought it was “safer”); and misunderstanding of the meaning of &quot;: many people seem to think it produces “smart quotes” (they apparently never looked at the actual results).

Anyway, there is never any need to use &quot; in element content in HTML (XHTML or any other HTML version). There is nothing in any HTML specification that would assign any special meaning to the plain character " there.

As the question says, it has its role in attribute values, but even in them, it is mostly simpler to just use single quotes as delimiters if the value contains a double quote, e.g. alt='Greeting: "Hello, World!"' or, if you are allowed to correct errors in natural language texts, to use proper quotation marks, e.g. alt="Greeting: “Hello, World!”"

Javascript add method to object

You can make bar a function making it a method.

Foo.bar = function(passvariable){  };

As a property it would just be assigned a string, data type or boolean

Foo.bar = "a place";

PHP-FPM and Nginx: 502 Bad Gateway

For anyone else struggling to get to the bottom of this, I tried adjusting timeouts as suggested as I didn't want to stop using Unix sockets...after lots of troubleshooting and not much to go on I found that this issue was being caused by the APC extension that I'd enabled in php-fpm a few months back. Disabling this extension resolved the intermittent 502 errors, easiest way to do this was by commenting out the following line :

;extension = apc.so

This did the trick for me!

How do I remove the blue styling of telephone numbers on iPhone/iOS?

Try using putting the ASCII character for the dash in between the digit separations.

from this: -

to this: &ndash;

ex: change 555-555-5555 => 555&ndash;555&ndash;5555

Can you require two form fields to match with HTML5?

As has been mentioned in other answers, there is no pure HTML5 way to do this.

If you are already using JQuery, then this should do what you need:

_x000D_
_x000D_
$(document).ready(function() {_x000D_
  $('#ourForm').submit(function(e){_x000D_
      var form = this;_x000D_
      e.preventDefault();_x000D_
      // Check Passwords are the same_x000D_
      if( $('#pass1').val()==$('#pass2').val() ) {_x000D_
          // Submit Form_x000D_
          alert('Passwords Match, submitting form');_x000D_
          form.submit();_x000D_
      } else {_x000D_
          // Complain bitterly_x000D_
          alert('Password Mismatch');_x000D_
          return false;_x000D_
      }_x000D_
  });_x000D_
});
_x000D_
<script src="https://cdnjs.cloudflare.com/ajax/libs/jquery/3.3.1/jquery.min.js"></script>_x000D_
<form id="ourForm">_x000D_
    <input type="password" name="password" id="pass1" placeholder="Password" required>_x000D_
    <input type="password" name="password" id="pass2" placeholder="Repeat Password" required>_x000D_
    <input type="submit" value="Go">_x000D_
</form>
_x000D_
_x000D_
_x000D_

Save a subplot in matplotlib

While @Eli is quite correct that there usually isn't much of a need to do it, it is possible. savefig takes a bbox_inches argument that can be used to selectively save only a portion of a figure to an image.

Here's a quick example:

import matplotlib.pyplot as plt
import matplotlib as mpl
import numpy as np

# Make an example plot with two subplots...
fig = plt.figure()
ax1 = fig.add_subplot(2,1,1)
ax1.plot(range(10), 'b-')

ax2 = fig.add_subplot(2,1,2)
ax2.plot(range(20), 'r^')

# Save the full figure...
fig.savefig('full_figure.png')

# Save just the portion _inside_ the second axis's boundaries
extent = ax2.get_window_extent().transformed(fig.dpi_scale_trans.inverted())
fig.savefig('ax2_figure.png', bbox_inches=extent)

# Pad the saved area by 10% in the x-direction and 20% in the y-direction
fig.savefig('ax2_figure_expanded.png', bbox_inches=extent.expanded(1.1, 1.2))

The full figure: Full Example Figure


Area inside the second subplot: Inside second subplot


Area around the second subplot padded by 10% in the x-direction and 20% in the y-direction: Full second subplot

Change date format in a Java string

Using the java.time package in Java 8 and later:

String date = "2011-01-18 00:00:00.0";
TemporalAccessor temporal = DateTimeFormatter
    .ofPattern("yyyy-MM-dd HH:mm:ss.S")
    .parse(date); // use parse(date, LocalDateTime::from) to get LocalDateTime
String output = DateTimeFormatter.ofPattern("yyyy-MM-dd").format(temporal);

Is there a native jQuery function to switch elements?

This is an answer based on @lotif's answer logic, but bit more generalized

If you append/prepend after/before the elements are actually moved
=> no clonning needed
=> events kept

There are two cases that can happen

  1. One target has something " .prev() ious" => we can put the other target .after() that.
  2. One target is the first child of it's .parent() => we can .prepend() the other target to parent.

The CODE

This code could be done even shorter, but I kept it this way for readability. Note that prestoring parents (if needed) and previous elements is mandatory.

$(function(){
  var $one = $("#one");
  var $two = $("#two");
  
  var $onePrev = $one.prev(); 
  if( $onePrev.length < 1 ) var $oneParent = $one.parent();

  var $twoPrev = $two.prev();
  if( $twoPrev.length < 1 ) var $twoParent = $two.parent();
  
  if( $onePrev.length > 0 ) $onePrev.after( $two );
    else $oneParent.prepend( $two );
    
  if( $twoPrev.length > 0 ) $twoPrev.after( $one );
    else $twoParent.prepend( $one );

});

...feel free to wrap the inner code in a function :)

Example fiddle has extra click events attached to demonstrate event preservation...
Example fiddle: https://jsfiddle.net/ewroodqa/

...will work for various cases - even one such as:

<div>
  <div id="one">ONE</div>
</div>
<div>Something in the middle</div>
<div>
  <div></div>
  <div id="two">TWO</div>
</div>

Load content of a div on another page

Yes, see "Loading Page Fragments" on http://api.jquery.com/load/.

In short, you add the selector after the URL. For example:

$('#result').load('ajax/test.html #container');

Finding the path of the program that will execute from the command line in Windows

Use the where command. The first result in the list is the one that will execute.

C:\> where notepad
C:\Windows\System32\notepad.exe
C:\Windows\notepad.exe

According to this blog post, where.exe is included with Windows Server 2003 and later, so this should just work with Vista, Win 7, et al.

On Linux, the equivalent is the which command, e.g. which ssh.

Ruby value of a hash key?

I didn't understand your problem clearly but I think this is what you're looking for(Based on my understanding)

  person =   {"name"=>"BillGates", "company_name"=>"Microsoft", "position"=>"Chairman"}

  person.delete_if {|key, value| key == "name"} #doing something if the key == "something"

  Output: {"company_name"=>"Microsoft", "position"=>"Chairman"}

Implementing multiple interfaces with Java - is there a way to delegate?

Unfortunately: NO.

We're all eagerly awaiting the Java support for extension methods

Install specific branch from github using Npm

I'm using SSH to authenticate my GitHub account and have a couple dependencies in my project installed as follows:

"dependencies": {
  "<dependency name>": "git+ssh://[email protected]/<github username>/<repository name>.git#<release version | branch>"
}

Remove android default action bar

I've noticed that if you set the theme in the AndroidManifest, it seems to get rid of that short time where you can see the action bar. So, try adding this to your manifest:

<android:theme="@android:style/Theme.NoTitleBar">

Just add it to your application tag to apply it app-wide.

jQuery UI - Draggable is not a function?

This issue can also be caused if you include the normal jquery library twice. I had the following line twice, in my body and head.

It never caused any problems until I tried to use jquery UI as well.

AngularJS: factory $http.get JSON file

Okay, here's a list of things to look into:

1) If you're not running a webserver of any kind and just testing with file://index.html, then you're probably running into same-origin policy issues. See:

https://code.google.com/archive/p/browsersec/wikis/Part2.wiki#Same-origin_policy

Many browsers don't allow locally hosted files to access other locally hosted files. Firefox does allow it, but only if the file you're loading is contained in the same folder as the html file (or a subfolder).

2) The success function returned from $http.get() already splits up the result object for you:

$http({method: 'GET', url: '/someUrl'}).success(function(data, status, headers, config) {

So it's redundant to call success with function(response) and return response.data.

3) The success function does not return the result of the function you pass it, so this does not do what you think it does:

var mainInfo = $http.get('content.json').success(function(response) {
        return response.data;
    });

This is closer to what you intended:

var mainInfo = null;
$http.get('content.json').success(function(data) {
    mainInfo = data;
});

4) But what you really want to do is return a reference to an object with a property that will be populated when the data loads, so something like this:

theApp.factory('mainInfo', function($http) { 

    var obj = {content:null};

    $http.get('content.json').success(function(data) {
        // you can do some processing here
        obj.content = data;
    });    

    return obj;    
});

mainInfo.content will start off null, and when the data loads, it will point at it.

Alternatively you can return the actual promise the $http.get returns and use that:

theApp.factory('mainInfo', function($http) { 
    return $http.get('content.json');
});

And then you can use the value asynchronously in calculations in a controller:

$scope.foo = "Hello World";
mainInfo.success(function(data) { 
    $scope.foo = "Hello "+data.contentItem[0].username;
});

Could not load file or assembly 'Newtonsoft.Json, Version=9.0.0.0, Culture=neutral, PublicKeyToken=30ad4fe6b2a6aeed' or one of its dependencies

Adding binding redirect configuration for Newtonsoft.Json in your configuration file (web.config) will resolve the issue.

<configuration>
    <runtime>
        <assemblyBinding xmlns="urn:schemas-microsoft-com:asm.v1">   
            <dependentAssembly>
                <assemblyIdentity name="Newtonsoft.Json" publicKeyToken="30ad4fe6b2a6aeed" culture="neutral" />
                <bindingRedirect oldVersion="0.0.0.0-12.0.0.0" newVersion="12.0.0.0" />
            </dependentAssembly>
        </assemblyBinding>
    </runtime>
</configuration>

Since Newtonsoft.Json version in your case is 9 update the version appropriatly in the configuration.

If this configuration does not work make sure the namespace (xmlns) in your configuration tag is correct or remove the name space completely.

Assembly binding redirect does not work

How-to turn off all SSL checks for postman for a specific site

enter image description here

click here in settings, one pop up window will get open. There we have switcher to make SSL verification certificate (Off)

Can you change what a symlink points to after it is created?

Yes, you can!

$ ln -sfn source_file_or_directory_name softlink_name

Oracle get previous day records

I think you can also execute this command:

select (sysdate-1) PREVIOUS_DATE from dual;

How do I get column names to print in this C# program?

Print datatable rows with column

Here is solution 

DataTable datatableinfo= new DataTable();

// Fill data table 

//datatableinfo=fill by function or get data from database

//Print data table with rows and column


for (int j = 0; j < datatableinfo.Rows.Count; j++)
{
    for (int i = 0; i < datatableinfo.Columns.Count; i++)    
        {    
            Console.Write(datatableinfo.Columns[i].ColumnName + " ");    
            Console.WriteLine(datatableinfo.Rows[j].ItemArray[i]+" "); 
        }
}


Ouput :
ColumnName - row Value
ColumnName - row Value
ColumnName - row Value
ColumnName - row Value

get keys of json-object in JavaScript

The working code

_x000D_
_x000D_
var jsonData = [{person:"me", age :"30"},{person:"you",age:"25"}];_x000D_
_x000D_
for(var obj in jsonData){_x000D_
    if(jsonData.hasOwnProperty(obj)){_x000D_
    for(var prop in jsonData[obj]){_x000D_
        if(jsonData[obj].hasOwnProperty(prop)){_x000D_
           alert(prop + ':' + jsonData[obj][prop]);_x000D_
        }_x000D_
    }_x000D_
}_x000D_
}
_x000D_
_x000D_
_x000D_

Matplotlib subplots_adjust hspace so titles and xlabels don't overlap?

The link posted by Jose has been updated and pylab now has a tight_layout() function that does this automatically (in matplotlib version 1.1.0).

http://matplotlib.org/api/pyplot_api.html#matplotlib.pyplot.tight_layout

http://matplotlib.org/users/tight_layout_guide.html#plotting-guide-tight-layout

How can I get the console logs from the iOS Simulator?

Download the safari technology review. With the simulator running, select develop > simulator > localhost

Is `shouldOverrideUrlLoading` really deprecated? What can I use instead?

Implement both deprecated and non-deprecated methods like below. First one is to handle API level 21 and higher, second one is handle lower than API level 21

webViewClient = object : WebViewClient() {
.
.
        @RequiresApi(Build.VERSION_CODES.LOLLIPOP)
        override fun shouldOverrideUrlLoading(view: WebView?, request: WebResourceRequest?): Boolean {
            parseUri(request?.url)
            return true
        }

        @SuppressWarnings("deprecation")
        override fun shouldOverrideUrlLoading(view: WebView?, url: String?): Boolean {
            parseUri(Uri.parse(url))
            return true
        }
}

How to access site running apache server over lan without internet connection

I was trying to access my localhost website (on my pc) from my mobile (andriod). The configuration is like Windows 10, WAMP 2.4.23, PHP Website and my mobile was running on andriod. Both my mobile and pc are connected to same wifi.

I was able to open my website on my pc by using url http://localhost/mysite or http://127.0.0.1/mysite. My pc ip was 192.168.0.1 (say) and my mobile ip was 192.168.0.2 (say) and both connected on same wifi.

I tried all the setting like changing the httpd.conf, httpd-vhosts.conf only to find that all I need was to disable my firewall. Of course, disabling the firewall completely is not a good idea. I have avast antivirus running on my pc. If I check the firewall log for last one hour (or so) I can see that attempt has been made by my mobile ip to connect to website running on my pc. All it required was to add an exception by creating a new rule in avast UI which will allow connections from my mobile ip.

Hope this helps someone.

Read line with Scanner

next() and nextLine() methods are associated with Scanner and is used for getting String inputs. Their differences are...

next() can read the input only till the space. It can't read two words separated by space. Also, next() places the cursor in the same line after reading the input.

nextLine() reads input including space between the words (that is, it reads till the end of line \n). Once the input is read, nextLine() positions the cursor in the next line.

Read article :Difference between next() and nextLine()

Replace your while loop with :

while(r.hasNext()) {
                scan = r.next();
                System.out.println(scan);
                if(scan.length()==0) {continue;}
                //treatment
            }

Using hasNext() and next() methods will resolve the issue.

How can I shrink the drawable on a button?

  • Using "BATCH DRAWABLE IMPORT" feature you can import custom size depending upon your requirement example 20dp*20dp

    • Now after importing use the imported drawable_image as drawable_source for your button

    • It's simpler this way enter image description here

Difference between dates in JavaScript

By using the Date object and its milliseconds value, differences can be calculated:

var a = new Date(); // Current date now.
var b = new Date(2010, 0, 1, 0, 0, 0, 0); // Start of 2010.
var d = (b-a); // Difference in milliseconds.

You can get the number of seconds (as a integer/whole number) by dividing the milliseconds by 1000 to convert it to seconds then converting the result to an integer (this removes the fractional part representing the milliseconds):

var seconds = parseInt((b-a)/1000);

You could then get whole minutes by dividing seconds by 60 and converting it to an integer, then hours by dividing minutes by 60 and converting it to an integer, then longer time units in the same way. From this, a function to get the maximum whole amount of a time unit in the value of a lower unit and the remainder lower unit can be created:

function get_whole_values(base_value, time_fractions) {
    time_data = [base_value];
    for (i = 0; i < time_fractions.length; i++) {
        time_data.push(parseInt(time_data[i]/time_fractions[i]));
        time_data[i] = time_data[i] % time_fractions[i];
    }; return time_data;
};
// Input parameters below: base value of 72000 milliseconds, time fractions are
// 1000 (amount of milliseconds in a second) and 60 (amount of seconds in a minute). 
console.log(get_whole_values(72000, [1000, 60]));
// -> [0,12,1] # 0 whole milliseconds, 12 whole seconds, 1 whole minute.

If you're wondering what the input parameters provided above for the second Date object are, see their names below:

new Date(<year>, <month>, <day>, <hours>, <minutes>, <seconds>, <milliseconds>);

As noted in the comments of this solution, you don't necessarily need to provide all these values unless they're necessary for the date you wish to represent.

Getting Integer value from a String using javascript/jquery

just do this , you need to remove char other than "numeric" and "." form your string will do work for you

yourString = yourString.replace ( /[^\d.]/g, '' );

your final code will be

  str1 = "test123.00".replace ( /[^\d.]/g, '' );
  str2 = "yes50.00".replace ( /[^\d.]/g, '' );
  total = parseInt(str1, 10) + parseInt(str2, 10);
  alert(total);

Demo

What is an idempotent operation?

Idempotence means that applying an operation once or applying it multiple times has the same effect.

Examples:

  • Multiplication by zero. No matter how many times you do it, the result is still zero.
  • Setting a boolean flag. No matter how many times you do it, the flag stays set.
  • Deleting a row from a database with a given ID. If you try it again, the row is still gone.

For pure functions (functions with no side effects) then idempotency implies that f(x) = f(f(x)) = f(f(f(x))) = f(f(f(f(x)))) = ...... for all values of x

For functions with side effects, idempotency furthermore implies that no additional side effects will be caused after the first application. You can consider the state of the world to be an additional "hidden" parameter to the function if you like.

Note that in a world where you have concurrent actions going on, you may find that operations you thought were idempotent cease to be so (for example, another thread could unset the value of the boolean flag in the example above). Basically whenever you have concurrency and mutable state, you need to think much more carefully about idempotency.

Idempotency is often a useful property in building robust systems. For example, if there is a risk that you may receive a duplicate message from a third party, it is helpful to have the message handler act as an idempotent operation so that the message effect only happens once.

List all employee's names and their managers by manager name using an inner join

SELECT DISTINCT e.Ename AS Employee, 
    m.mgr AS reports_to, 
    m.Ename AS manager 
FROM Employees e, Employees m 
WHERE e.mgr=m.EmpID;

How do I append to a table in Lua

I'd personally make use of the table.insert function:

table.insert(a,"b");

This saves you from having to iterate over the whole table therefore saving valuable resources such as memory and time.

Why use getters and setters/accessors?

If you want a readonly variable but don't want the client to have to change the way they access it, try this templated class:

template<typename MemberOfWhichClass, typename primative>                                       
class ReadOnly {
    friend MemberOfWhichClass;
public:
    template<typename number> inline bool   operator==(const number& y) const { return x == y; } 
    template<typename number> inline number operator+ (const number& y) const { return x + y; } 
    template<typename number> inline number operator- (const number& y) const { return x - y; } 
    template<typename number> inline number operator* (const number& y) const { return x * y; }  
    template<typename number> inline number operator/ (const number& y) const { return x / y; } 
    template<typename number> inline number operator<<(const number& y) const { return x << y; }
    template<typename number> inline number operator^(const number& y) const  { return x^y; }
    template<typename number> inline number operator~() const                 { return ~x; }
    template<typename number> inline operator number() const                  { return x; }
protected:
    template<typename number> inline number operator= (const number& y) { return x = y; }       
    template<typename number> inline number operator+=(const number& y) { return x += y; }      
    template<typename number> inline number operator-=(const number& y) { return x -= y; }      
    template<typename number> inline number operator*=(const number& y) { return x *= y; }      
    template<typename number> inline number operator/=(const number& y) { return x /= y; }      
    primative x;                                                                                
};      

Example Use:

class Foo {
public:
    ReadOnly<Foo, int> cantChangeMe;
};

Remember you'll need to add bitwise and unary operators as well! This is just to get you started

Globally catch exceptions in a WPF application?

Use the Application.DispatcherUnhandledException Event. See this question for a summary (see Drew Noakes' answer).

Be aware that there'll be still exceptions which preclude a successful resuming of your application, like after a stack overflow, exhausted memory, or lost network connectivity while you're trying to save to the database.

Proper way to renew distribution certificate for iOS

Very simple was to renew your certificate. Go to your developer member centre and go to your Provisioning profile and see what are the certificate Active and InActive and select Inactive certificate and hit Edit button then hit generate button. Now your certificate successful renewal for another 1 year. Thanks

How to configure log4j.properties for SpringJUnit4ClassRunner?

I was using Maven in eclipse and I did not want to have an additional copy of the properties file in the root folder. You can do the following in eclipse:

  1. Open run dialog (click the little arrow next to the play button and go to run configurations)
  2. Go to the "classpath" tab
  3. Select the "User Entries" and click the "Advanced" button on the right side.
  4. Now select the "Add External folder" radio button.
  5. Select the resources folder

converting Java bitmap to byte array

Try something like this:

Bitmap bmp = intent.getExtras().get("data");
ByteArrayOutputStream stream = new ByteArrayOutputStream();
bmp.compress(Bitmap.CompressFormat.PNG, 100, stream);
byte[] byteArray = stream.toByteArray();
bmp.recycle();

Absolute and Flexbox in React Native

This solution worked for me:

tabBarOptions: {
      showIcon: true,
      showLabel: false,
      style: {
        backgroundColor: '#000',
        borderTopLeftRadius: 40,
        borderTopRightRadius: 40,
        position: 'relative',
        zIndex: 2,
        marginTop: -48
      }
  }

How do you round a double in Dart to a given degree of precision AFTER the decimal point?

var price = 99.012334554;
price = price.toStringAsFixed(2);
print(price); // 99.01

That is the ref of dart. ref: https://api.dartlang.org/stable/2.3.0/dart-core/num/toStringAsFixed.html

Trees in Twitter Bootstrap

If someone wants expandable/collapsible version of the treeview from Vitaliy Bychik's answer, you can save some time :)

http://jsfiddle.net/mehmetatas/fXzHS/2/

$(function () {
    $('.tree li').hide();
    $('.tree li:first').show();
    $('.tree li').on('click', function (e) {
        var children = $(this).find('> ul > li');
        if (children.is(":visible")) children.hide('fast');
        else children.show('fast');
        e.stopPropagation();
    });
});

How to retrieve the current value of an oracle sequence without increment it?

If your use case is that some backend code inserts a record, then the same code wants to retrieve the last insert id, without counting on any underlying data access library preset function to do this, then, as mentioned by others, you should just craft your SQL query using SEQ_MY_NAME.NEXTVAL for the column you want (usually the primary key), then just run statement SELECT SEQ_MY_NAME.CURRVAL FROM dual from the backend.

Remember, CURRVAL is only callable if NEXTVAL has been priorly invoked, which is all naturally done in the strategy above...

What is a simple command line program or script to backup SQL server databases?

I found this on a Microsoft Support page http://support.microsoft.com/kb/2019698.

It works great! And since it came from Microsoft, I feel like it's pretty legit.

Basically there are two steps.

  1. Create a stored procedure in your master db. See msft link or if it's broken try here: http://pastebin.com/svRLkqnq
  2. Schedule the backup from your task scheduler. You might want to put into a .bat or .cmd file first and then schedule that file.

    sqlcmd -S YOUR_SERVER_NAME\SQLEXPRESS -E -Q "EXEC sp_BackupDatabases @backupLocation='C:\SQL_Backup\', @backupType='F'"  1>c:\SQL_Backup\backup.log            
    

Obviously replace YOUR_SERVER_NAME with your computer name or optionally try .\SQLEXPRESS and make sure the backup folder exists. In this case it's trying to put it into c:\SQL_Backup

How to show the text on a ImageButton?

ImageButton can't have text (or, at least, android:text isn't listed in its attributes).

The Trick is:

It looks like you need to use Button (and look at drawableTop or setCompoundDrawablesWithIntrinsicBounds(int,int,int,int)).

How to get the background color code of an element in hex?

You have the color you just need to convert it into the format you want.

Here's a script that should do the trick: http://www.phpied.com/rgb-color-parser-in-javascript/

Check if a file exists or not in Windows PowerShell?

The standard way to see if a file exists is with the Test-Path cmdlet.

Test-Path -path $filename

Specific Time Range Query in SQL Server

you can try this (I don't have sql server here today so I can't verify syntax, sorry)

select attributeName
  from tableName
 where CONVERT(varchar,attributeName,101) BETWEEN '03/01/2009' AND '03/31/2009'
   and CONVERT(varchar, attributeName,108) BETWEEN '06:00:00' AND '22:00:00'
   and DATEPART(day,attributeName) BETWEEN 2 AND 4

How to pass all arguments passed to my bash script to a function of mine?

Use the $@ variable, which expands to all command-line parameters separated by spaces.

abc "$@"

How do I get the current time zone of MySQL?

It may be

select timediff(current_time(),utc_time())

You won't get the timezone value directly this way.

@@global.time_zone cannot be used as it is a variable, and it returns the value 'SYSTEM'.

If you need to use your query in a session with a changed timezone by using session SET TIME_ZONE =, then you will get that with @@session.time_zone. If you query @@global.time_zone, then you get 'SYSTEM'.

If you try datediff, date_sub, or timediff with now() and utc_time(), then you'll probably run into conversion issues.

But the things suggested above will probably work at least with some server versions. My version is 5.5.43-37 and is a hosted solution.

Remove leading and trailing spaces?

You can use the strip() to remove trailing and leading spaces.

>>> s = '   abd cde   '
>>> s.strip()
'abd cde'

Note: the internal spaces are preserved

What does "@" mean in Windows batch scripts

you can include @ in a 'scriptBlock' like this:

@(
  echo don't echoed
  hostname
)
echo echoed

and especially do not do that :)

for %%a in ("@") do %%~aecho %%~a

C++ convert string to hexadecimal and vice versa

A string like "Hello World" to hex format: 48656C6C6F20576F726C64.

Ah, here you go:

#include <string>

std::string string_to_hex(const std::string& input)
{
    static const char hex_digits[] = "0123456789ABCDEF";

    std::string output;
    output.reserve(input.length() * 2);
    for (unsigned char c : input)
    {
        output.push_back(hex_digits[c >> 4]);
        output.push_back(hex_digits[c & 15]);
    }
    return output;
}

#include <stdexcept>

int hex_value(unsigned char hex_digit)
{
    static const signed char hex_values[256] = {
        -1, -1, -1, -1, -1, -1, -1, -1, -1, -1, -1, -1, -1, -1, -1, -1,
        -1, -1, -1, -1, -1, -1, -1, -1, -1, -1, -1, -1, -1, -1, -1, -1,
        -1, -1, -1, -1, -1, -1, -1, -1, -1, -1, -1, -1, -1, -1, -1, -1,
         0,  1,  2,  3,  4,  5,  6,  7,  8,  9, -1, -1, -1, -1, -1, -1,
        -1, 10, 11, 12, 13, 14, 15, -1, -1, -1, -1, -1, -1, -1, -1, -1,
        -1, -1, -1, -1, -1, -1, -1, -1, -1, -1, -1, -1, -1, -1, -1, -1,
        -1, 10, 11, 12, 13, 14, 15, -1, -1, -1, -1, -1, -1, -1, -1, -1,
        -1, -1, -1, -1, -1, -1, -1, -1, -1, -1, -1, -1, -1, -1, -1, -1,
        -1, -1, -1, -1, -1, -1, -1, -1, -1, -1, -1, -1, -1, -1, -1, -1,
        -1, -1, -1, -1, -1, -1, -1, -1, -1, -1, -1, -1, -1, -1, -1, -1,
        -1, -1, -1, -1, -1, -1, -1, -1, -1, -1, -1, -1, -1, -1, -1, -1,
        -1, -1, -1, -1, -1, -1, -1, -1, -1, -1, -1, -1, -1, -1, -1, -1,
        -1, -1, -1, -1, -1, -1, -1, -1, -1, -1, -1, -1, -1, -1, -1, -1,
        -1, -1, -1, -1, -1, -1, -1, -1, -1, -1, -1, -1, -1, -1, -1, -1,
        -1, -1, -1, -1, -1, -1, -1, -1, -1, -1, -1, -1, -1, -1, -1, -1,
        -1, -1, -1, -1, -1, -1, -1, -1, -1, -1, -1, -1, -1, -1, -1, -1,
    };
    int value = hex_values[hex_digit];
    if (value == -1) throw std::invalid_argument("invalid hex digit");
    return value;
}

std::string hex_to_string(const std::string& input)
{
    const auto len = input.length();
    if (len & 1) throw std::invalid_argument("odd length");

    std::string output;
    output.reserve(len / 2);
    for (auto it = input.begin(); it != input.end(); )
    {
        int hi = hex_value(*it++);
        int lo = hex_value(*it++);
        output.push_back(hi << 4 | lo);
    }
    return output;
}

(This assumes that a char has 8 bits, so it's not very portable, but you can take it from here.)

HTML5 iFrame Seamless Attribute

Use the frameborder attribute on your iframe and set it to frameborder="0" . That produces the seamless look. Now you maybe saying I want the nested iframe to control rather I have scroll bars. Then you need to whip up a JavaScript script file that calculates height minus any headers and set the height. Debounce is javascript plugin needed to make sure resize works appropriately in older browsers and sometimes chrome. That will get you in the right direction.

Why does fatal error "LNK1104: cannot open file 'C:\Program.obj'" occur when I compile a C++ project in Visual Studio?

I had this exact error when building a VC++ DLL in Visual Studio 2019:

LNK1104: cannot open file 'C:\Program.obj'

Turned out under project Properties > Linker > Input > Module Definition File, I had specified a def file that had an unmatched double-quote at the end of the filename. Deleting the unmatched double quote resolved the issue.

How to set a transparent background of JPanel?

Calling setOpaque(false) on the upper JPanel should work.

From your comment, it sounds like Swing painting may be broken somewhere -

First - you probably wanted to override paintComponent() rather than paint() in whatever component you have paint() overridden in.

Second - when you do override paintComponent(), you'll first want to call super.paintComponent() first to do all the default Swing painting stuff (of which honoring setOpaque() is one).

Example -

import java.awt.Color;
import java.awt.Graphics;

import javax.swing.JFrame;
import javax.swing.JPanel;


public class TwoPanels {
    public static void main(String[] args) {

        JPanel p = new JPanel();
        // setting layout to null so we can make panels overlap
        p.setLayout(null);

        CirclePanel topPanel = new CirclePanel();
        // drawing should be in blue
        topPanel.setForeground(Color.blue);
        // background should be black, except it's not opaque, so 
        // background will not be drawn
        topPanel.setBackground(Color.black);
        // set opaque to false - background not drawn
        topPanel.setOpaque(false);
        topPanel.setBounds(50, 50, 100, 100);
        // add topPanel - components paint in order added, 
        // so add topPanel first
        p.add(topPanel);

        CirclePanel bottomPanel = new CirclePanel();
        // drawing in green
        bottomPanel.setForeground(Color.green);
        // background in cyan
        bottomPanel.setBackground(Color.cyan);
        // and it will show this time, because opaque is true
        bottomPanel.setOpaque(true);
        bottomPanel.setBounds(30, 30, 100, 100);
        // add bottomPanel last...
        p.add(bottomPanel);

        // frame handling code...
        JFrame f = new JFrame("Two Panels");
        f.setContentPane(p);
        f.setDefaultCloseOperation(JFrame.EXIT_ON_CLOSE);
        f.setSize(300, 300);
        f.setLocationRelativeTo(null);
        f.setVisible(true);
    }

    // Panel with a circle drawn on it.
    private static class CirclePanel extends JPanel {

        // This is Swing, so override paint*Component* - not paint
        protected void paintComponent(Graphics g) {
            // call super.paintComponent to get default Swing 
            // painting behavior (opaque honored, etc.)
            super.paintComponent(g);
            int x = 10;
            int y = 10;
            int width = getWidth() - 20;
            int height = getHeight() - 20;
            g.drawArc(x, y, width, height, 0, 360);
        }
    }
}

Using number_format method in Laravel

If you are using Eloquent the best solution is:

public function getFormattedPriceAttribute()
{
    return number_format($this->attributes['price'], 2);
}

So now you must append formattedPrice in your model and you can use both, price (at its original state) and formattedPrice.

How to set time zone in codeigniter?

add it in your index.php file, and it will work on all over your site

if ( function_exists( 'date_default_timezone_set' ) ) {
    date_default_timezone_set('Asia/Kolkata');
}

How to refresh a page with jQuery by passing a parameter to URL

Click these links to see these more flexible and robust solutions. They're answers to a similar question:

These allow you to programmatically set the parameter, and, unlike the other hacks suggested for this question, won't break for URLs that already have a parameter, or if something else isn't quite what you thought might happen.

What is Python buffer type for?

I think buffers are e.g. useful when interfacing python to native libraries. (Guido van Rossum explains buffer in this mailinglist post).

For example, numpy seems to use buffer for efficient data storage:

import numpy
a = numpy.ndarray(1000000)

the a.data is a:

<read-write buffer for 0x1d7b410, size 8000000, offset 0 at 0x1e353b0>

.NET Console Application Exit Event

As a good example may be worth it to navigate to this project and see how to handle exiting processes grammatically or in this snippet from VM found in here

                ConsoleOutputStream = new ObservableCollection<string>();

                var startInfo = new ProcessStartInfo(FilePath)
                {
                    WorkingDirectory = RootFolderPath,
                    Arguments = StartingArguments,
                    RedirectStandardOutput = true,
                    UseShellExecute = false,
                    CreateNoWindow = true
                };
                ConsoleProcess = new Process {StartInfo = startInfo};

                ConsoleProcess.EnableRaisingEvents = true;

                ConsoleProcess.OutputDataReceived += (sender, args) =>
                {
                    App.Current.Dispatcher.Invoke((System.Action) delegate
                    {
                        ConsoleOutputStream.Insert(0, args.Data);
                        //ConsoleOutputStream.Add(args.Data);
                    });
                };
                ConsoleProcess.Exited += (sender, args) =>
                {
                    InProgress = false;
                };

                ConsoleProcess.Start();
                ConsoleProcess.BeginOutputReadLine();
        }
    }

    private void RegisterProcessWatcher()
    {
        startWatch = new ManagementEventWatcher(
            new WqlEventQuery($"SELECT * FROM Win32_ProcessStartTrace where ProcessName = '{FileName}'"));
        startWatch.EventArrived += new EventArrivedEventHandler(startProcessWatch_EventArrived);

        stopWatch = new ManagementEventWatcher(
            new WqlEventQuery($"SELECT * FROM Win32_ProcessStopTrace where ProcessName = '{FileName}'"));
        stopWatch.EventArrived += new EventArrivedEventHandler(stopProcessWatch_EventArrived);
    }

    private void stopProcessWatch_EventArrived(object sender, EventArrivedEventArgs e)
    {
        InProgress = false;
    }

    private void startProcessWatch_EventArrived(object sender, EventArrivedEventArgs e)
    {
        InProgress = true;
    }

What does body-parser do with express?

Yes we can work without body-parser. When you don't use that you get the raw request, and your body and headers are not in the root object of request parameter . You will have to individually manipulate all the fields.

Or you can use body-parser, as the express team is maintaining it .

What body-parser can do for you: It simplifies the request.
How to use it: Here is example:

Install npm install body-parser --save

This how to use body-parser in express:

const express = require('express'),
      app = express(),
      bodyParser = require('body-parser');

// support parsing of application/json type post data
app.use(bodyParser.json());

//support parsing of application/x-www-form-urlencoded post data
app.use(bodyParser.urlencoded({ extended: true }));

Link.

https://github.com/expressjs/body-parser.

And then you can get body and headers in root request object . Example

app.post("/posturl",function(req,res,next){
    console.log(req.body);
    res.send("response");
})

How to get the onclick calling object?

The thing with your method is that you clutter your HTML with javascript. If you put your javascript in an external file you can access your HTML unobtrusive and this is much neater.

Lateron you can expand your code with addEventListener/attackEvent(IE) to prevent memory leaks.

This is without jQuery

<a href="123.com" id="elementid">link</a>

window.onload = function () {
  var el = document.getElementById('elementid');
  el.onclick = function (e) {
    var ev = e || window.event;
    // here u can use this or el as the HTML node
  }
}

You say you want to manipulate it with jQuery. So you can use jQuery. Than it is even better to do it like this:

// this is the window.onload startup of your JS as in my previous example. The difference is 
// that you can add multiple onload functions
$(function () {
  $('a#elementid').bind('click', function (e) {
    // "this" points to the <a> element
    // "e" points to the event object
  });
});

Loop through all elements in XML using NodeList

    DocumentBuilderFactory dbf = DocumentBuilderFactory.newInstance();
    DocumentBuilder db = dbf.newDocumentBuilder();
    Document dom = db.parse("file.xml");
    Element docEle = dom.getDocumentElement();
    NodeList nl = docEle.getChildNodes();
    int length = nl.getLength();
    for (int i = 0; i < length; i++) {
        if (nl.item(i).getNodeType() == Node.ELEMENT_NODE) {
            Element el = (Element) nl.item(i);
            if (el.getNodeName().contains("staff")) {
                String name = el.getElementsByTagName("name").item(0).getTextContent();
                String phone = el.getElementsByTagName("phone").item(0).getTextContent();
                String email = el.getElementsByTagName("email").item(0).getTextContent();
                String area = el.getElementsByTagName("area").item(0).getTextContent();
                String city = el.getElementsByTagName("city").item(0).getTextContent();
            }
        }
    }

Iterate over all children and nl.item(i).getNodeType() == Node.ELEMENT_NODE is used to filter text nodes out. If there is nothing else in XML what remains are staff nodes.

For each node under stuff (name, phone, email, area, city)

 el.getElementsByTagName("name").item(0).getTextContent(); 

el.getElementsByTagName("name") will extract the "name" nodes under stuff, .item(0) will get you the first node and .getTextContent() will get the text content inside.

Edit: Since we have jackson I would do this in a different way. Define a pojo for the object:

public class Staff {
    private String name;
    private String phone;
    private String email;
    private String area;
    private String city;
...getters setters
}

Then using jackson:

    JsonNode root = new XmlMapper().readTree(xml.getBytes());
    ObjectMapper mapper = new ObjectMapper();
    root.forEach(node -> consume(node, mapper));



private void consume(JsonNode node, ObjectMapper mapper) {
    try {
        Staff staff = mapper.treeToValue(node, Staff.class);
        //TODO your job with staff
    } catch (JsonProcessingException e) {
        e.printStackTrace();
    }
}

What is a "static" function in C?

Minimal runnable multi-file scope example

Here I illustrate how static affects the scope of function definitions across multiple files.

a.c

#include <stdio.h>

/* Undefined behavior: already defined in main.
 * Binutils 2.24 gives an error and refuses to link.
 * https://stackoverflow.com/questions/27667277/why-does-borland-compile-with-multiple-definitions-of-same-object-in-different-c
 */
/*void f() { puts("a f"); }*/

/* OK: only declared, not defined. Will use the one in main. */
void f(void);

/* OK: only visible to this file. */
static void sf() { puts("a sf"); }

void a() {
    f();
    sf();
}

main.c

#include <stdio.h>

void a(void);        

void f() { puts("main f"); }

static void sf() { puts("main sf"); }

void m() {
    f();
    sf();
}

int main() {
    m();
    a();
    return 0;
}

GitHub upstream.

Compile and run:

gcc -c a.c -o a.o
gcc -c main.c -o main.o
gcc -o main main.o a.o
./main

Output:

main f
main sf
main f
a sf

Interpretation

  • there are two separate functions sf, one for each file
  • there is a single shared function f

As usual, the smaller the scope, the better, so always declare functions static if you can.

In C programming, files are often used to represent "classes", and static functions represent "private" methods of the class.

A common C pattern is to pass a this struct around as the first "method" argument, which is basically what C++ does under the hood.

What standards say about it

C99 N1256 draft 6.7.1 "Storage-class specifiers" says that static is a "storage-class specifier".

6.2.2/3 "Linkages of identifiers" says static implies internal linkage:

If the declaration of a file scope identifier for an object or a function contains the storage-class specifier static, the identifier has internal linkage.

and 6.2.2/2 says that internal linkage behaves like in our example:

In the set of translation units and libraries that constitutes an entire program, each declaration of a particular identifier with external linkage denotes the same object or function. Within one translation unit, each declaration of an identifier with internal linkage denotes the same object or function.

where "translation unit" is a source file after preprocessing.

How GCC implements it for ELF (Linux)?

With the STB_LOCAL binding.

If we compile:

int f() { return 0; }
static int sf() { return 0; }

and disassemble the symbol table with:

readelf -s main.o

the output contains:

Num:    Value          Size Type    Bind   Vis      Ndx Name
  5: 000000000000000b    11 FUNC    LOCAL  DEFAULT    1 sf
  9: 0000000000000000    11 FUNC    GLOBAL DEFAULT    1 f

so the binding is the only significant difference between them. Value is just their offset into the .bss section, so we expect it to differ.

STB_LOCAL is documented on the ELF spec at http://www.sco.com/developers/gabi/2003-12-17/ch4.symtab.html:

STB_LOCAL Local symbols are not visible outside the object file containing their definition. Local symbols of the same name may exist in multiple files without interfering with each other

which makes it a perfect choice to represent static.

Functions without static are STB_GLOBAL, and the spec says:

When the link editor combines several relocatable object files, it does not allow multiple definitions of STB_GLOBAL symbols with the same name.

which is coherent with the link errors on multiple non static definitions.

If we crank up the optimization with -O3, the sf symbol is removed entirely from the symbol table: it cannot be used from outside anyways. TODO why keep static functions on the symbol table at all when there is no optimization? Can they be used for anything?

See also

C++ anonymous namespaces

In C++, you might want to use anonymous namespaces instead of static, which achieves a similar effect, but further hides type definitions: Unnamed/anonymous namespaces vs. static functions

Check string for palindrome

I'm new to java and I'm taking up your question as a challenge to improve my knowledge.

import java.util.ArrayList;
import java.util.List;

public class PalindromeRecursiveBoolean {

    public static boolean isPalindrome(String str) {

        str = str.toUpperCase();
        char[] strChars = str.toCharArray();

        List<Character> word = new ArrayList<>();
        for (char c : strChars) {
            word.add(c);
        }

        while (true) {
            if ((word.size() == 1) || (word.size() == 0)) {
                return true;
            }
            if (word.get(0) == word.get(word.size() - 1)) {
                word.remove(0);
                word.remove(word.size() - 1);
            } else {
                return false;

            }

        }
    }
}
  1. If the string is made of no letters or just one letter, it is a palindrome.
  2. Otherwise, compare the first and last letters of the string.
    • If the first and last letters differ, then the string is not a palindrome
    • Otherwise, the first and last letters are the same. Strip them from the string, and determine whether the string that remains is a palindrome. Take the answer for this smaller string and use it as the answer for the original string then repeat from 1.

nginx missing sites-available directory

If you'd prefer a more direct approach, one that does NOT mess with symlinking between /etc/nginx/sites-available and /etc/nginx/sites-enabled, do the following:

  1. Locate your nginx.conf file. Likely at /etc/nginx/nginx.conf
  2. Find the http block.
  3. Somewhere in the http block, write include /etc/nginx/conf.d/*.conf; This tells nginx to pull in any files in the conf.d directory that end in .conf. (I know: it's weird that a directory can have a . in it.)
  4. Create the conf.d directory if it doesn't already exist (per the path in step 3). Be sure to give it the right permissions/ownership. Likely root or www-data.
  5. Move or copy your separate config files (just like you have in /etc/nginx/sites-available) into the directory conf.d.
  6. Reload or restart nginx.
  7. Eat an ice cream cone.

Any .conf files that you put into the conf.d directory from here on out will become active as long as you reload/restart nginx after.

Note: You can use the conf.d and sites-enabled + sites-available method concurrently if you wish. I like to test on my dev box using conf.d. Feels faster than symlinking and unsymlinking.

How to confirm RedHat Enterprise Linux version?

I assume that you've run yum upgrade. That will in general update you to the newest minor release.

Your main resources for determining the version are /etc/redhat_release and lsb_release -a

implementing merge sort in C++

I know this question has already been answered, but I decided to add my two cents. Here is code for a merge sort that only uses additional space in the merge operation (and that additional space is temporary space which will be destroyed when the stack is popped). In fact, you will see in this code that there is not usage of heap operations (no declaring new anywhere).

Hope this helps.

    void merge(int *arr, int size1, int size2) {
        int temp[size1+size2];
        int ptr1=0, ptr2=0;
        int *arr1 = arr, *arr2 = arr+size1;

        while (ptr1+ptr2 < size1+size2) {
            if (ptr1 < size1 && arr1[ptr1] <= arr2[ptr2] || ptr1 < size1 && ptr2 >= size2)
                temp[ptr1+ptr2] = arr1[ptr1++];

            if (ptr2 < size2 && arr2[ptr2] < arr1[ptr1] || ptr2 < size2 && ptr1 >= size1)
                temp[ptr1+ptr2] = arr2[ptr2++];
        }   

        for (int i=0; i < size1+size2; i++)
            arr[i] = temp[i];
    }   

    void mergeSort(int *arr, int size) {
        if (size == 1)
            return;

        int size1 = size/2, size2 = size-size1;
        mergeSort(arr, size1);
        mergeSort(arr+size1, size2);
        merge(arr, size1, size2);
    } 

    int main(int argc, char** argv) {
         int num;
         cout << "How many numbers do you want to sort: ";
         cin >> num;
         int a[num];
         for (int i = 0; i < num; i++) {
           cout << (i + 1) << ": ";
           cin >> a[i];
         }   

         // Start merge sort
         mergeSort(a, num);

         // Print the sorted array
         cout << endl;
         for (int i = 0; i < num; i++) {
           cout << a[i] << " ";
         }   
         cout << endl;

         return 0;
    } 

Does MS SQL Server's "between" include the range boundaries?

The BETWEEN operator is inclusive.

From Books Online:

BETWEEN returns TRUE if the value of test_expression is greater than or equal to the value of begin_expression and less than or equal to the value of end_expression.

DateTime Caveat

NB: With DateTimes you have to be careful; if only a date is given the value is taken as of midnight on that day; to avoid missing times within your end date, or repeating the capture of the following day's data at midnight in multiple ranges, your end date should be 3 milliseconds before midnight on of day following your to date. 3 milliseconds because any less than this and the value will be rounded up to midnight the next day.

e.g. to get all values within June 2016 you'd need to run:

where myDateTime between '20160601' and DATEADD(millisecond, -3, '20160701')

i.e.

where myDateTime between '20160601 00:00:00.000' and '20160630 23:59:59.997'

datetime2 and datetimeoffset

Subtracting 3 ms from a date will leave you vulnerable to missing rows from the 3 ms window. The correct solution is also the simplest one:

where myDateTime >= '20160601' AND myDateTime < '20160701'

change html input type by JS?

This is not supported by some browsers (IE if I recall), but it works in the rest:

document.getElementById("password-field").attributes["type"] = "password";

or

document.getElementById("password-field").attributes["type"] = "text";

Vertically centering Bootstrap modal window

As of Bootstrap 4 the following class was added to achieve this for you without JavaScript.

modal-dialog-centered

You can find their documentation here.

Thanks v.d for pointing out you need to add both .modal-dialog-centered to .modal-dialog to vertically center the modal.

Creating a blurring overlay view

I decided to post a written Objective-C version from the accepted answer just to provide more options in this question..

- (UIView *)applyBlurToView:(UIView *)view withEffectStyle:(UIBlurEffectStyle)style andConstraints:(BOOL)addConstraints
{
  //only apply the blur if the user hasn't disabled transparency effects
  if(!UIAccessibilityIsReduceTransparencyEnabled())
  {
    UIBlurEffect *blurEffect = [UIBlurEffect effectWithStyle:style];
    UIVisualEffectView *blurEffectView = [[UIVisualEffectView alloc] initWithEffect:blurEffect];
    blurEffectView.frame = view.bounds;

    [view addSubview:blurEffectView];

    if(addConstraints)
    {
      //add auto layout constraints so that the blur fills the screen upon rotating device
      [blurEffectView setTranslatesAutoresizingMaskIntoConstraints:NO];

      [view addConstraint:[NSLayoutConstraint constraintWithItem:blurEffectView
                                                       attribute:NSLayoutAttributeTop
                                                       relatedBy:NSLayoutRelationEqual
                                                          toItem:view
                                                       attribute:NSLayoutAttributeTop
                                                      multiplier:1
                                                        constant:0]];

      [view addConstraint:[NSLayoutConstraint constraintWithItem:blurEffectView
                                                       attribute:NSLayoutAttributeBottom
                                                       relatedBy:NSLayoutRelationEqual
                                                          toItem:view
                                                       attribute:NSLayoutAttributeBottom
                                                      multiplier:1
                                                        constant:0]];

      [view addConstraint:[NSLayoutConstraint constraintWithItem:blurEffectView
                                                       attribute:NSLayoutAttributeLeading
                                                       relatedBy:NSLayoutRelationEqual
                                                          toItem:view
                                                       attribute:NSLayoutAttributeLeading
                                                      multiplier:1
                                                        constant:0]];

      [view addConstraint:[NSLayoutConstraint constraintWithItem:blurEffectView
                                                       attribute:NSLayoutAttributeTrailing
                                                       relatedBy:NSLayoutRelationEqual
                                                          toItem:view
                                                       attribute:NSLayoutAttributeTrailing
                                                      multiplier:1
                                                        constant:0]];
    }
  }
  else
  {
    view.backgroundColor = [[UIColor blackColor] colorWithAlphaComponent:0.7];
  }

  return view;
}

The constraints could be removed if you want incase if you only support portrait mode or I just add a flag to this function to use them or not..

Python convert object to float

I eventually used:

weather["Temp"] = weather["Temp"].convert_objects(convert_numeric=True)

It worked just fine, except that I got the following message.

C:\ProgramData\Anaconda3\lib\site-packages\ipykernel_launcher.py:3: FutureWarning:
convert_objects is deprecated.  Use the data-type specific converters pd.to_datetime, pd.to_timedelta and pd.to_numeric.

.crx file install in chrome

I arrived to this question looking for the same but for Chromium (actually I'm using https://ungoogled-software.github.io). So in case anyone else is looking for the same:

  1. Go to chrome://flags/
  2. Search for Handling of extension MIME type requests
  3. Select Always prompt for install
  4. Search for an extension and copy its URL (something like https://chrome.google.com/webstore/detail/...)
  5. Paste the URL in https://crxextractor.com/ and download the .CRX
  6. Voilà, Chromium will prompt for installation

mongodb how to get max value from collections

The performance of the suggested answer is fine. According to the MongoDB documentation:

When a $sort immediately precedes a $limit, the optimizer can coalesce the $limit into the $sort. This allows the sort operation to only maintain the top n results as it progresses, where n is the specified limit, and MongoDB only needs to store n items in memory.

Changed in version 4.0.

So in the case of

db.collection.find().sort({age:-1}).limit(1)

we get only the highest element WITHOUT sorting the collection because of the mentioned optimization.

How can I disable ReSharper in Visual Studio and enable it again?

You need to goto Tools-->Options--->Select Resharper--->Click on suspend now,to disable it

Call a "local" function within module.exports from another function in module.exports?

const Service = {
  foo: (a, b) => a + b,
  bar: (a, b) => Service.foo(a, b) * b
}

module.exports = Service

Find all packages installed with easy_install/pip?

The below is a little slow, but it gives a nicely formatted list of packages that pip is aware of. That is to say, not all of them were installed "by" pip, but all of them should be able to be upgraded by pip.

$ pip search . | egrep -B1 'INSTALLED|LATEST'

The reason it is slow is that it lists the contents of the entire pypi repo. I filed a ticket suggesting pip list provide similar functionality but more efficiently.

Sample output: (restricted the search to a subset instead of '.' for all.)

$ pip search selenium | egrep -B1 'INSTALLED|LATEST'

selenium                  - Python bindings for Selenium
  INSTALLED: 2.24.0
  LATEST:    2.25.0
--
robotframework-selenium2library - Web testing library for Robot Framework
  INSTALLED: 1.0.1 (latest)
$

Difference between static and shared libraries?

For a static library, the code is extracted from the library by the linker and used to build the the final executable at the point you compile/build your application. The final executable has no dependencies on the library at run time

For a shared library, the compiler/linker checks that the names you link with exist in the library when the application is built, but doesn't move their code into the application. At run time, the shared library must be available.

The C programming language itself has no concept of either static or shared libraries - they are completely an implementation feature.

Personally, I much prefer to use static libraries, as it makes software distribution simpler. However, this is an opinion over which much (figurative) blood has been shed in the past.

How to write a full path in a batch file having a folder name with space?

CD E:\Documents and Settings\All Users\Application Data

E:\Documents and Settings\All Users\Application Data>REGSVR32 xyz.dll

Compare integer in bash, unary operator expected

Judging from the error message the value of i was the empty string when you executed it, not 0.

HTML5 Dynamically create Canvas

Alternative

Use element .innerHTML= which is quite fast in modern browsers

_x000D_
_x000D_
document.body.innerHTML = "<canvas width=500 height=150 id='CursorLayer'>";


// TEST

var ctx = CursorLayer.getContext("2d");
ctx.fillStyle = "red";
ctx.fillRect(100, 100, 50, 50);
_x000D_
canvas { border: 1px solid black }
_x000D_
_x000D_
_x000D_

Java, return if trimmed String in List contains String

Try this:

for(String str: myList) {
    if(str.trim().equals("A"))
       return true;
}
return false;

You need to use str.equals or str.equalsIgnoreCase instead of contains because contains in string works not the same as contains in List

List<String> s = Arrays.asList("BAB", "SAB", "DAS");
s.contains("A"); // false
"BAB".contains("A"); // true

filter: progid:DXImageTransform.Microsoft.gradient is not working in ie7

You didn't specify a GradientType:

background: #f0f0f0; /* Old browsers */
background: -moz-linear-gradient(top, #ffffff 0%, #eeeeee 100%); /* FF3.6+ */
background: -webkit-gradient(linear, left top, left bottom, color-stop(0%,#ffffff), color-stop(100%,#eeeeee)); /* Chrome,Safari4+ */
background: -webkit-linear-gradient(top, #ffffff 0%,#eeeeee 100%); /* Chrome10+,Safari5.1+ */
background: -o-linear-gradient(top, #ffffff 0%,#eeeeee 100%); /* Opera11.10+ */
background: -ms-linear-gradient(top, #ffffff 0%,#eeeeee 100%); /* IE10+ */
filter: progid:DXImageTransform.Microsoft.gradient( startColorstr='#ffffff', endColorstr='#eeeeee',GradientType=0 ); /* IE6-9 */
background: linear-gradient(top, #ffffff 0%,#eeeeee 100%); /* W3C */

source: http://www.colorzilla.com/gradient-editor/

Eclipse: Java was started but returned error code=13

Since you didn't mention the version of Eclipse, I advice you to download the latest version of Eclipse Luna which comes with Java 8 support by default.

Is it better practice to use String.format over string Concatenation in Java?

About performance:

public static void main(String[] args) throws Exception {      
  long start = System.currentTimeMillis();
  for(int i = 0; i < 1000000; i++){
    String s = "Hi " + i + "; Hi to you " + i*2;
  }
  long end = System.currentTimeMillis();
  System.out.println("Concatenation = " + ((end - start)) + " millisecond") ;

  start = System.currentTimeMillis();
  for(int i = 0; i < 1000000; i++){
    String s = String.format("Hi %s; Hi to you %s",i, + i*2);
  }
  end = System.currentTimeMillis();
  System.out.println("Format = " + ((end - start)) + " millisecond");
}

The timing results are as follows:

  • Concatenation = 265 millisecond
  • Format = 4141 millisecond

Therefore, concatenation is much faster than String.format.

How to resolve "Input string was not in a correct format." error?

If using TextBox2.Text as the source for a numeric value, it must first be checked to see if a value exists, and then converted to integer.

If the text box is blank when Convert.ToInt32 is called, you will receive the System.FormatException. Suggest trying:

protected void SetImageWidth()
{
   try{
      Image1.Width = Convert.ToInt32(TextBox1.Text);
   }
   catch(System.FormatException)
   {
      Image1.Width = 100; // or other default value as appropriate in context.
   }
}

Clear contents of cells in VBA using column reference

I just came up with this very simple method of clearing an entire sheet.

Sub ClearThisSheet()

ActiveSheet.UsedRange.ClearContents

End Sub

Searching if value exists in a list of objects using Linq

Another possibility

if (list.Count(customer => customer.Firstname == "John") > 0) {
 //bla
}

Can angularjs routes have optional parameter values?

It looks like Angular has support for this now.

From the latest (v1.2.0) docs for $routeProvider.when(path, route):

path can contain optional named groups with a question mark (:name?)

Open button in new window?

You can acheive this using window.open() method, passing _blank as one of the parameter. You can refer the below links which has more information on this.

http://www.w3schools.com/jsref/met_win_open.asp

http://msdn.microsoft.com/en-us/library/ms536651(v=vs.85).aspx

Hope this will help you.

Has Windows 7 Fixed the 255 Character File Path Limit?

You can get around that limit by using subst if you need to.

Sending POST parameters with Postman doesn't work, but sending GET parameters does

I faced the same issue in PostMan and Advance REST Client both. I checked through fiddler and found that my request payload is not converted into JSON format.

I am passing my data in Body as x-www-form-urlencoded enter image description here

You can fix it by using Content-Type as application/x-www-form-urlencoded in request header. enter image description here

How to check for registry value using VbScript

Try something like this:

Dim windowsShell
Dim regValue
Set windowsShell = CreateObject("WScript.Shell")
regValue = windowsShell.RegRead("someRegKey")

MySQL - Trigger for updating same table after insert

On the last entry; this is another trick:

SELECT AUTO_INCREMENT FROM information_schema.tables WHERE table_schema = ... and table_name = ...